Sunteți pe pagina 1din 303
of DIFFERENTIAL EQUATIONS | by FRANK AYRES, JR _ including * 560 solved problems Completely Solved in Detail e j i SCHAUM’S OUTLINE SERIES Py McGRAW-HILL BOOK COMPANY ? SCHAUM’S OUTLINE OF THEORY AND PROBLEMS OF DIFFERENTIAL EQUATIONS BY FRANK AYRES, JR., Ph.D. Professor and Head, Department of Mathematics Dickinson College SCHAUYPS OUTLINE SERIES McGRAW-HILL BOOK COMPANY New York, St. Louis, San Francisco, Toronto, Sydney Copyright © 1952 by McGraw-Hill Inc. Alt Rights Reserved. Printed in the United States of America. No part of this publication may be reproduced, stored in a retrieval system, or transmitted, in any form or by any means, electronic, nrechanical, photocopying, recording, or otherwise, without the prior written permission of the publisher. 02654 4567890 SHSH 721069 Preface This book is designed primarily to supplement standard texts in elementary differential equations. All types of ordinary and partial differential equations found in current texts. together with the various procedures for solving them. are included. Since the beginning student must be concerned largely with mastering the methods of solving a variety of different type equations, it is felt that there is need for a comprehensive problem book sueh as this. It should prove also of equal service to practicing engineers and scientists who feel the need for a review of the theory and problem work in this increasingly important field. Each chapter. except for the third whiel: is entirely expository. begins with a brief statement of definitions. principles. and theorems. followed by a set of solved and supplementary problems. These solved problems have been selected to make a careful study of each as rewarding as possible. Equal attention has been given to the chapters on applications. which include a wide variety of problems from geometry and the physical sciences. Much more taterial is presented here than can be taken up in most first courses. This is done not only to meet any choice of toptes which the instructor may make. but also to stimulate further interest in the subject and to provide a handy book of referenee. However. this book is definitely not a formal text- book and. sinee there is always a tendency to “get on” with the problems, those being introduced to the subject for the first time are warned against using it as a means of avoiding a thorough stndy of the regular text. The author is pleased to acknowledge his indebtedness to Mr. Louis Sand- ler. associate editor of the publishers. for invaluable suggestions and critical. review of the entire manuscript. FRANK Ayrks. JR. Carlisle, Pa. September, 1952 — - CHAPTER PAGE J. ORIGIN OF DIFFERENTIAL EQUATIONS. cccccsccssecssssssssssecssscssecsssecsessssssessssssssterensentsnestensesszes 1] 2. SOLUTIONS OF DIFFERENTIAL EQUATIONS sisson . 7 3. EQuaTIONS oF First ORDER AND First DEGREE cseseenn sscsnesecssestenssesessenstecccssees 12 4. EQuaTions oF First ORDER AND First DEGREE — VARIABLES SEPA- RABLE AND REDUCTION TO VARIABLES SEPARABLE oovccscssccssmsssseessuneesee 15 5. EQuatTIONs oF First ORDER AND First DEGREE — Exact EQUATIONS AND REDUCTION TO EXACT EQUATIONS | ccssssssssssssssssssssssusssessssesessssunsasssssisssesssates 24 6. Equations or First ORDER AND First DEGREE — LuvEAR EQuATIONS AND THOSE REDUCIBLE TO THAT FORM csssssssssscecsssctssssscessscsssesseesaseecseressecseses 35 T. GEOMETRIG APPLICATIONS cscsesssssscsssssssssssscssssssssesssssssesscecsssssceesenssecssususssssesenssssssssssunsensgeecessera 4] 8. PrysicaL APPLICATIONS ... ssessscensvssssstesuecenensansuassnsncensansnesenescoseensnssa desssasetusassessssanstessusssess wu 49 9. EQuaATIONS OF FIRST ORDER AND HIGHER DEGREF ossssssssssnssssssssgsseninscscee 61 10. SINGULAR SOLUTIONS — EXTRANEOUS LOCI csnssssssssscsscemeseensesstscssercecersnsneestnett 67 11. AppLicaTiONs OF First ORDER AND HIGHER DEGREE EQUATIONS ....0.0% 75 12. LINEAR EQUATIONS OF ORDER TD ccsssssssssssesseesesnessssinessssessciussicescteensnssssssscessessssnnstt 718 13. HomocENrKous LINEAR EQUATIONS WITH CONSTANT COEFFICIENTS we 82 14. Linear Equations WITH CONSTANT COEFFICIENTS 06 ssessssssaase sesssaaeena sone BT 15. Linear Equations wirh CONSTANT COEFFICIENTS — VARIATION OF PARAMETERS, UNDETERMINED COEFFICIENTS. cssssssssssssssnesessee usin 93 16. Linear Eouations with ConsTANT COEFFICIENTS — SHORT METHODS ween sassesssestesanenesnsctsgeuncsntess sessccssuuecsevssssssssessusavesscesnussccccs saseeesssesaeeee sesccsststessnsecsstsssnserronee 99 17. Linear Equations: wiTH VARIABLE COEFFICIENTS — THE CAUCHY AND LEGENDRE LINEAR EQUATIONS ssssscssnnsusnmennaniamusonsnnnansennenene 108 18. Linear EQuaTions wiTH VARIABLE COEFFICIENTS — EQUATIONS OF Contents essa ernasnaaeeineeneanevasnns LIL] THE SECOND ORDER uissieeen sesvansssnsseensesse sessssvsee sessssevesessee sas CHAPTER PAGE 19. Linear EQuATIONS WITH VARIABLE COEFFICIENTS — MisCELLANEOUS TYPES ceecccsssosssscsesccessesstecseseees seousssestsussteconsesceeceeestsereessssreeene esesssascncansussaseecessarsarsasassseseeeseeeeeerotenate wee 122 20. APPLICATIONS OF LINEAR EQUATIONS sissies 133 2]. SYSTEMS OF SIMULTANEOUS LINEAR EQUATIONS cccsssssvssesssscsssscssscsstsessssnssestanee 157 22. Tora DIFFERENTIAL EQUATIONS. ccssssssssssssssccssssseseesesesessnanstetsssssssssssssenasssvecseeststssssgnnees 161 23. APPLICATIONS OF TOTAL AND SIMULTANEOUS EQUATIONS ccscsssssseseesseesees 178 24. NUMERICAL APPROXIMATIONS TO SOLUTIONS cesesssssssssecsssssssmesnsressensessssrcntetenses 186 25. INTEGRATION IN SERIES 26. INTEGRATION IN SERIES 27. THE LEGENDRE,. BESSEL. AND GAUSS EQUATIONS cssssssssesssesesesscsscmesessenentenees 220 28. PARTIAL DIFFERENTIAL EQUATIONS. cescsssssssssssssecsssssscesecsssssccecscssssssecssssessecssecessnveeceserteesaes 23) 29. LINEAR PARTIAL DIFFERENTIAL EQUATIONS OF ORDER ONE cress 238 30. Non-LINEAR ParTiAL DIFFERENTIAL EQUATIONS OF ORDER ONE. uses 244 31. Homocrenrous PartiAL DIFFERENTIAL EQUATIONS OF HIGHER ORDER WITH CONSTANT COEFFICIENTS. cccssssssssssssssssssssssssssssvssesseessecsssssssssssssssssessssarsssnsensesesseneenss 255 32. NON-HOMOGENKOUS LINEAR EQUATIONS WITH CONSTANT COEKFFI- CIENTS eccscssscsesssscscesescsseessessssenssessssssersecansssssesssssessessvssssssusasssatoessssunsssvessesessevassceseesensesecseseeeseeenseenstetesess 265 33. Partial DIFFERENTIAL EQUATIONS OF ORDER Two witit VARIABLE COEFFICIENTS —cssssssssscssssscossssesssussssesesssssessnssssesssussasvusssssstosesssssssseseeseseagsuussssustustassssssssssesssssseeeuces 276 CHAPTER 1 Origin of Differential Equations A DIFFERENTIAL EQUATION is an equation which involves derivatives. For example, d 1) Saxes 5) (y")? + (yt)? + By = x? dx d? d 3 3 2) CX 43% + ay = 0 6) 2 = 2 + xe dx? dx Ox ‘Oy ' - 2 2 3) xylty 3 7) oz + o2 = x? + y, Bx? oy? 4) y" + 2(y")? + y! = cos x If there is a single independent variabie, as in 1)-5), the derivatives are ordinary derivatives and the equation is called an ordinary differential equa- tion, If there are two or more independent variables, as in 6)-7), the deriva- tives are partial derivatives and the equation is called a partial differen- tial equation, The order of a differential equation is the order of the highest derivative which occurs. Equations 1), 3), and 6) are of the first order; 2), 5), and 7) are of the second order; and 4) is of the third order. The degree of a differential equation which can be written as a polynomial in the derivatives is the degree of the highest ordered derivative which then occurs. All of the above examples are of the first degree except 5) which is of the second degree. A discussion of partial differential equations will be given in Chapter 28. For the present, only ordinary differential equations with a single dependent variable will be considered. ORIGIN OF DIFFERENTIAL EQUATIONS. a) Geometric Problems. See Problems 1 and 2 below. b) Physical Problems. See Problems 2 and 4 below. c) Primitives. A relation between the variables which involves n essential arbitrary conStants, as y= x*tC€x or y = Ax?+Bx, is called a primitive. The n constants, always indicated by capital letters here, are called essen- tial if they cannot be replaced by a smaller number of constants. See Prob- lem 5. In general, a primitive involving n essential arbitrary constants will give rise to a differential equation, of order n, free of arbitrary constants. This equation is obtained by eliminating the n constants between the (n +1) equa- tions consisting of the primitive and the n equations obtained by differen- tiating the primitive n times with respect to the independent variable. See Problems 6-14 below. 1. 6. ORIGIN OF DIFFERENTIAL EQUATIONS SOLVED PROBLEMS A curve is defined by the condition that at each of its points (x,y), its slope dy/dx is equal to twice the sum of the coordinates of the point. Express the condition by means of a differential equation, da: The differential equation representing the condition is ~ = 2(x+y). A curve is defined by the condition that the sum of the x- andy-intercepts of its tangents is always equal to 2, Express the condition by means of a differential equation, d The equation of the tangent at (x,y) on the curve is Y-y = SX = 2) and the x- and y- intercepts are respectively X = x-y = and Y = y-x 2, The differential equation represent- d ing the condition is X + Y =x-yS + y¥-% =2 or «Qy -(x+ye- a2 +y=0. oy dx One hundred grams of cane sugar in water are being converted into dextrose at a rate which is proportional to the amount unconverted. Find the differential equation expressing the rate of conversion after t minutes, Let gq denote the number of grams converted in t minutes, Then (100-—q) is thenumber of grams unconverted and the rate of conversion is given by a = k(100-q), k being the constant of proportionality. A particle of mass m moves along a straight line (the x-axis) while subject to 1) a force pro- portional to its displacement x from a fixed point O in its path and directed toward O and 2) a resisting force proportional to its velocity. Express thetotal forceas a differential equa- tion. The first force may be represented by —k,x and the second by kn » where k, and ky are factors of proportionality. d*x dx The total force (mass x acceleration) is given by m™ => ~kyx -— ky a t dt In each of the equations a) y.= x?+A+B, b) y= Aen *? c) y = A+ 1nBx show that only one of the two arbitrary constants is essential. a) Since A+B is no more than a single arbitrary constant, only one essential arbitrary con- stant is involved. +B B b) y = Ae~ = Ae*e , and Ae® is no more than a single arbitrary constant, ce) y = A+ In Bx = A+ 1nB+ 1n«x, and (A + 1n B) is no more than a single constant. Obtain the differential equation associated with the primitive y = Ax? + Bx +C. Since there are three arbitrary constants, we consider the four equations 2 3 y = Ax? + Bus, ay _ oar +B, dy - 4, dY 9, dx dx? dx} 3 d The last of these =, being free of arbitrary constants and of the proper order, is the dx 10. ORIGIN OF DIFFERENTIAL EQUATIONS required equation. Note that the constants could not have been eliminated between the first threeof the above equations. Note also that the primitive can be obtained readily from the differential equa- tion by integration, Obtain the differential equation associated with the primitive xy? + wy = C, dy Differentiating once with respect to x, we obtain (2xy?+ ay?) + (3x? yP+ Bary? =) = 0 or, when xy #0, (2y + 3x ) + xy’ (By + 5x 2) = 0 as the required equation. When written in differential notation, these equations are 1) (2xy?dx + 3x7y?dy) + (3x?y?dx + Sx? y*dy) = 0 and 2) (2y dx + 3xdy) + xy’ (3y dx + Sxdy) = 0. Note that the primitive can be obtained readily from 1) by integration but not so readily from 2). Thus, to obtain the primitive when 2) is given, it is necessary to determine the fac-~ tor xy? which was removed from 1). Obtain the differential equation associated with the primitive y = A cos ax + B sin ax, A and B being arbitrary constants, and a being a fixed constant, dy . Here — = -—Aa sin ax + Ba cos ax dx d? and oY = ~Aa® cos ax - Ba? sin ax = —a? (A cos ax + B sin ax) = -a’y, dx? a? The required differential equation is <2 + a’y = 0, dx Obtain the differential equation associated with the primitive y = Ae” +Be*+C. d d° d° Here oY De? + Be, oY = ghe®* + BeX, -3 = gAe** + Be*, dx dx? dx 3 2 2 3 2 2 then 2Y ~2Y 2 gaeex 4Y _ WY _ on ex ang CY _ FY - Ad _ Y), dx? dx? dx? ax dx} dx? dx? dx 3 2 The required equation is dy 392% , 9M] 0. dx} dx? dx Obtain the differential equation associated with the primitive y = Cye* + Coe* + Cae". d: 2 a 2x Here = = 3C,e* + 2Cze * Cae, <2 = 9C,e* + 4Coe” + Cge’, Q 3 and = = 27C,e* + 8Coe* + Cge. The elimination of the constants by elementary methods is somewhat tedious, If three of the equations are solved for C,, C,, €,, using determinants, and these substituted in the fourth equation, the result may be put in the form (called the eliminant): 11. 12. 13. ORIGIN OF DIFFERENTIAL EQUATIONS e* e* e y 111 sy 3e°* 2e* e% yr wf 3821 Y! , 2x = eI =e (~2y" + 12y" — 2ayi + Lay) = 0, ge“ 4e" e* y” 9 4 1 y" aie” ge* @% yl" 27 8 1 ¥" 3 2 The required differential equation is dy - 6 dy + 11 dy - 6y = 0. dx? dx? dx Obtain the differential equation associated with the primitive y = Cx? + Cc. . dy 1 dy 2 2 1 dy 2 1 dy.2 Since — = 2Cx, C= — —= and = Cx" + Co = — = xo + —~(=). de Ox dx yee ax de® * 72 de . . . . . dy 2 3 dy 2 The required differential equation is rm + 2x xz - 4x y = 0. Note, The primitive involves one arbitrary constant of degree two and the resulting dif- ferential equation is of order one and degree two. Find the differential equation of the family of circles of fixed radius r withcenters on the x-axis, The equation of the family is (x -Cy* + y? = 7? C being an arbitrary constant. Then (x-C) + y a = 0, x-C=-y 2, and the differential equation is vB + y = ory, Find the differential equation of the family of parabolas with foci at the origin and axes along the x-axis, x? +y? = (QA +x) y? = 4A(A+ x) y* = 4A(A+x) The equation of the family of parabolas is y? = 4A(A + x). Then yy’ = 24, A-=4yy', and y? = 2yy"(Syy! +x). dy 2, ox Y _y = 0. The required equation is = qa yn oh 14. 15. 16. ORIGIN OF DIFFERENTIAL EQUATIONS Form the differential equation representing all tangents to the parabola y = 2x. At any point (A,B) on the parabola, the equation of the tangent is y-—B = (x-—A)/B or, since A = 4B, By = x + 4B, Eliminating B between this and By’=1, obtained by differenti- ation with respect to x, we have as the required differential equation Qx(y')? ~ 2yy'+1=0. SUPPLEMENTARY PROBLEMS Classify each of the following equations as to order and degree. a) dy + (xy ~ cos x)dx = 0 Ans. Order one; degree one 2 by LER RB, 0 Ans. Order two; degree one dt? dt C c) y+ xy + ayy!) + xy =0 Ans, Order three; degree one d°v dv d d) =~ — + x(a)? +v=0 Ans, Order two; degree one dx? dx. dx 3 2 e) (ty? ~ (vy + vw = 0 Ans, Order three; degree two dv? dv? fy e - xy" +y=0 Ans. Order three; degree does not apply g) vo'+pe = sin O Ans, Order one; degree one 3 hy yl +x" = (y-xy'y” Ans. Order one; degree four 2 4 L) ap = fot (ey Ans. Order two; degree four d@2 dé Write the differential equation for each of the curves determined by the given conditions. a) At each point (x,y) the slope of the tangent is equal to the square of the abscissa of the point. Ans. y! = x? b) At each point (x,y) the length of the subtangent is equal to the sum of the coordinates of the point. Ans. y/y' = x+y or (x+y)yl ay. c) The segment joining P(x,y) and the point of intersection of the normal at P with the x-axis is bisected by the y-axis. Ans. y+ ct = sy or yy! + 2x=0 d) At each point (9,6) the tangent of the angle between the radius vector and the tangent is 1 equal to 1/3 the tangent of the vectorial angle. Ans. p « 3 tan 6 lo e) The area bounded by the arc of a curve, the x-axis, and two ordinates, one fixed and one variable, is equal to twice the length of the arc between the ordinates, x % Hint: [ova = af Avo ax. Ans. ¥ = 2+ (y'* a a Uw 17. 18. 19. 20. 21. 22. ORIGIN OF DIFFERENTIAL EQUATIONS Express each of the following physical statements in differential equation form, a) Radium decomposes at a rate proportional to the amount Q present. Ans. dQ/dt = —kQ b) The population P of a city increases at a rate proportional to the population and to the difference between 200,000 and the population, Ans. dP/dt = kP(200,000 - P) c) For a certain substance the rate of change of vapor pressure (P) with respect to temper- ature (7) is proportional to the vapor pressure and inversely proportional to the square of the temperature. Ans. dP/dT = kP/T? d) The potential difference F across an element of inductance L is equal to the product of L and the time rate of change of the current i in the inductance. Ans. E = L a . dv . d’s e) Mass x acceleration = net force. Ans. m— =F or m= F dt dt? Obtain the differential equation associated with the given primitive, A and B being arbitrary constants, a) y = Ax Ans. y! = y/x e) y = sin(x+A) Ans, yy =l- y? b) y= Ax +B Ans. y" = 0 fy y=de +B Ans, y" = y! c) y-e** = Be® Ans. y! = y g) x =Asin(y+B) Ans. y" = x(y'y d) y=Asinx Ans. y! = y cotx h) In y = Ax’? +B Ans. xyy" - yy! = x(y') = 0 Find the differential equation of the family of circles of variable radii r with centers on the x-axis, (Compare with Problem 12.) Hint: (x-A)*+y? = r?, A andr being arbitrary constants. Ans. yy" + (ty +1=0 Find the differential equation of the family of cardiods p.= a(1- cos @), Ans. (1 - cos@)do = p sin 6 dé Find the differential equation of all straight lines at a unit distance from the origin. Ans. (zy! yy? ze 1l¢+ wyty Find the differential equation of all circles in the plane. Hint: Use x? 4 y? ~2Ax~2By+C = 0. Ans. [1 ty) Jy” - By'(y") = 0 CHAPTER 2 Solutions of Differential Equations THE PROBLEM in elementary differential equations is essentially that of recovering the primitive which gave rise to the equation. In other words, the problem of solving a differential equation of order n is essentially that of finding a relation between the variables involving n independent arbitrary constants which together with the derivatives obtained fromit satisfy the differential equation. For example: Differential Equation Primitive d’y > 1)—- =0 y =Ax* + Bx +C (Prob.6, Chap. 1) dx) d*y d’y dy _ 30 2x x 2) ao 62 + 11 - 6¥=0 y=C,e + Cle + Cre (Prob.10, Chap. 1) 3) y( By? + y= 1? (x-C)? + y= r? (Prob.12, Chap. 1) THE CONDITIONS under which we can be assured thata differential equation is solv- able are given by Existence Theorems. For example, a differential equation of the form y'= g(x,y) for which a) g(x,y) is continuous and single valued over a region R of points (x,y), b) oe exists and is continuous at all points in R, y admits an infinity of solutions f(x,y,C) =0 (C, an arbitrary constant) such that through each point of R there passes one and only one curve of the fan- ily f(x,y,C) =0. See Problem 5. A PARTICULAR SOLUTION of a differential equation is one obtained from the primitive by assigning definite values to the arbitrary constants. For example, in 1) above y=0 (A4=B=C=0), y=2x+5 (A=0, B=2, C=5), and y=x?+2x+3 (A =], B=2, C=8) are particular solutions. Geometrically, the primitive is the equation of a family of curves and a particular solution is the equation of some one of the curves. These curves are called integral curves of the differential equation. As will be seen from Problem 6, a given form of the primitive may not in- clude all of the particular solutions. Moreover, as will be seen from Prob- lem 7, a differential equation may have solutions which cannot be obtained from the primitive by any manipulation of the arbitrary constant as in Problem 6, Such solutions, called singular solutions, will be considered in Chapter 10. The primitive of a differential equation is usually called the general so- dution of the equation. Certain authors, because of the remarks in the para- graph above, call it a general solution of the equation, SOLUTIONS OF DIFFERENTIAL EQUATIONS A DIFFERENTIAL EQUATION x = g(x,y) associates with each point (x 9,y¥ 9) in the re- gion R of the above existence theorem a direction m = dy = §(X9,¥o)- Ax | Gos Yo) 1. The direction at each such point is that of the tangent to the curve of the family f(x,y,C) =0, that is, the primitive, passing through the point. The region R with the direction at each of its points indicated is called a direc- tion field. In the adjoining figure, a num- ber of points with the direction at each is shown for the equation dy/dx = 2x. The in- tegral curves of the differential equation are those curves having at each of their points the direction given by the equation. In this example, the integral curves are parabolas. 8 $ : ® + x Such diagrams are helpful in that they -2 3 aid in clarifying the relation between a Q ¢ differential equation and its primitive, but since the integral curves are generally b 8 p ¢ = : = Slope = -4 4 \ \\ slope =4 quite complex, such a diagram does not aid lope = <2 hone «2 Slope = -! Slope = materially in obtaining their equations. slope <0 SOLVED PROBLEMS Show by direct substitution in the differential equation and a check of the arbitrary constants that each primitive gives rise to the corresponding differential equation. dy dy a) y = Cysinx + Cox (l-x cotx)-~= -x=+y = 0 dx? dx x x 2% @ d* d' x b) y = Cye’ + Coxe” + Cge + 2x e oy _ oY Ds y = Be dx? dx? dx d d* a) Substitute y = C, sinx + Cox, =C, cosx + Co, a = -~C,sinx inthe differential equa- It tion to obtain (1 - x cotx)(-C, sinx) - x(C, cosx + Cg) + (Cy sinx + Cgx) = ~C, sinx + C,x cosx -— C,x cosx — Cox + Cy, sinx+Cgx = 0. The order of the differential equation (2) and the number of arbitrary constants (2) agree. x b) y= Cye" + Coxe~ + Cye"* + 2x7 e*, (C, + Ca)ye™ + Coxe™ - C3e~ + 2x? e* + 4xe™, Se K + y" = (Cy+2Cy)e* + Coxe” Cye~ + axe” + gre* + 4e*, y" = (Cy+3Cy)e~ + Coxe” - Cge™~ + ax7e* + 12xe% + 12e”, 2 2 3 and y”- y"— y' + y = 8e. The order of the differential equation and the number of arbitrary constants agree, Show that y = 2x+ Ce~ is the primitive of the differential equation 2 —y = 2(1-x) and find xX the particular solution satisfied by x=0, y=3 (t.e., the equation of the integral curve through (0,3). 3. > SOLUTIONS OF DIFFERENTIAL EQUATIONS 9 Substitute y = 2x+Ce” and 2 = 2+Ce™ in the differential equation to obtain 2+ Ce — (2x+ Ce") = 2-2x, When x=0, y=3, 3=2:0+Ce° and C=3, The particular solution is y = 2x+ Be”. 2 d Show that y = Cye*+ Coee* +x is the primitive of the differential equation <2 - 3% + 2y = dx 2x —-3 and find the equation of the integral curve through the points (0,0) and (1,0). 2 Substitute y = Cye%+ Coe “+x, 2 = Cye“+ 2Cye* +1, fy =Cye*+4C,e°” in the dif- ferential equation to obtain Cyen+ 4Coe°* — 3(Cye* + 2Coe* +1) + 2(Cye~+ Coe +x) = 2x-3. When x =0, y=0: C,+(Cyg = 0. When x= 1, y= 0: Cye+ Coe? = -l, x 2x Then C, = -C, = } and the required equation is y=x+ 27". e?- é€ e? -¢€ 2 . ays . ; : dy,2 dy Show that (y-C)=Cx is the primitive of the differential equation ax (7) + x -y = 0 Xe and find the equations of the integral curves through the point (1,2). dy dy Cc Here 2(y-C)=-=C and =~ = ——~—. (YO) da 2-0) 2 2 2 2 Then 4x—C Oy - CHG -Dayy-G) 2 "Gag _ 4(y-C)? 2(y -C) (y -C)? (y -C) When x=1, y=2: (2-C)?=C and C=1,4, The equations of the integral curves through (1,2) are (y - 1)? =x and (y - 4)? = 4x. The primitive of the differential equation is y = Cx, Find the equation of the in- & 1S N RIS tegral curve through a) (1,2) and 6) (0,0). a) When x =1, y=2: C=2 and the required equation is y = 2x. 6) When x =0, y=0: C is not determined, that is, all of the integral curves pass through the origin, Note that g(x,y) = y/x is not continuous at the origin andhence theexistence theorem assures one and only one curve of the family y = Cx through each point of the plane except the origin, Differentiating xy = C(x-1)(y-1) and substituting for C, we obtain the differential equa- tion dy dy xy dy Xm + = Cle -1)— + -1 = x-1)— + -i1 y { 7 y -1} Gogo a y-1} dy 1 -1)2 -1) = 0 or ) X(x a + ¥(y-1) 0 Now both y=0 and y=1 are solutions of 1), since, for each, dy/dx=0 and 1) is satisfied. The first is obtained from the primitive by setting C=0, but the second y=1 cannot be obtained by assigning a finite value to C. Similarly, 1) may be obtained from the primitive in the form Bry = (x -1)(y-1). Now the solution y=1 is obtained by setting B=0 while the solution y =0 cannot be obtained by assigning a finite value to B. Thus, the given form of a primitive may not include all of the particular solutions of the differential equation. (Note that x=1 is also a particular solution, ) 10 SOLUTIONS OF DIFFERENTIAL EQUATIONS 2 a {. Differentiating y = Cx+2C°, solving for C = = » and substituting inthe primitive yields the differential equation dy.2 dy 1 2(—-) + =) - = 0. ) ree x() y . 1 Since y = ~ ae 2 =o i satisfies 1), a+ 8y = 0 is a solution of 1). Now the primitive is represented by a family of straight lines and it is clear that the equation of a parabola cannot be obtained by manipulating the arbitrary constant. Such a solu- tion is called a singular solution of the differential equation, 8. Verify and reconcile the fact that y = Cycosx + Cosinx and y =A cos(x+B) are primitives 2 of yay = 0. dx? From y = C, cosx+ Cy sinx, = -C, sinx + C,cosx and 2 = -C,cosx~Cygsinx = -y or ay y= 0, y! i y ? iu From y = Acos(x+B), y' = -Asin(x+B) and y”= -Acos(x+B) = -y. Now y = Acos(x+B) = A(cosx cosB —~ sinx sinB) = (A cosB)cosx + (-AsinB) sinx = C,cosx + Cysinx, 2 x 9. Show that In x? + In& = A+x may be written as y* = Be™. x 2 ? 2 ? 2 2 A+x A x x In x *inX = In ) = ny = At+x, Then y = e = ere = Be, x x 10. Show that Arc sinx -— Arc siny = A may be written as aV1-—y? - ¥ 1-x? = B, sincArc sin x - Arc sin y) = sinA = B, Then sin(Are sin x) cos(Are sin y) — cos(Arc sin x) sin( Are sin y) = xV1-y? -yV1-x? =B, 11. Show that In(1+y) + In(1+x) =A may be written as xy+x+y =C. In(l+y) + In(l+x) = In(l+y)(1+z) =A, Then (1+ y)(l+x) = xytxt+y+1=e4 =B and xyt+xt+y =B-1=C. 12. Show that sinh y + coshy = Cx may be written as y= Inx + A, Here sinhy + coshy = S(e’-e -) + d(e%+e”) = e = Cx, Then y = InC+Inx = A+ Inx, SOLUTIONS OF DIFFERENTIAL EQUATIONS J SUPPLEMENTARY PROBLEMS Show that each of the following expressions is a solution of the corresponding differential equation. Classify each as a particular solution or general solution (primitive). 13. yrex, xy! = 2y. Particular solution 2 2 say 14. x +y =, yyl'+x = 0. Primitive 15. y = Cx + c’, y = xy! + yt). Primitive 2. 3 Bo 2 2 . . 16. (l-x)y =x, Qx“y! = y(y + 3x). Particular solution 1% oy = e“(1+x), y" = 2y'+y=0. Particular solution 18. y = Cyx + Coe”, (x-ljy" - xy' + y= 0. General solution 19 y= Cye” + Coe”, y"-y = 0. General solution 20. y = Cye” + Coe +x - 4, yv-yr4-K, General solution Ql. y = Cye™ + Coe, y" — By! + 2y = 0. General solution 2e*(1-x). General solution Wl 22. ¥ Cye~ + Cae°* + xe”, y" - By! + 2y tt | CHAPTER 3 Equations of First Order and First Degree A DIFFERENTIAL EQUATION of the first order and first degree may be written in the form 1) M(x,y)dx + N(x,y)dy = 0. EXAMPLE 1. a) 2 + r= = 0 may be written as (y +x)dx + (y-x)dy =0 in which H(x,y) =yt+x and N(x,y)=y-x. b) dy _ Ltx’y may be written as (1+ x’y)dx —dy =0 in which dx M(x,y)=1+x’y and N¢x,y)=-1. If 4(x,y)dx+N(x,y)dy is the complete differential of a function p“(x,y), that i i at is, if W(x, y)dx + N(x,y) dy = du(x,y). 1) is called exact and u(x,y) =C is its primitive or general solution. EXAMPLE 2. 3x" y* dx + 2x’ y dy=0 is an exact differential equation since 3x’ y? dx + 2x3 y dy = d(x’y’). Its primitive is x’y’ =C. If 1) is not exact but E(x,y){M(x,y)dx + N(x,y)dy} = du(x,y), &(x,y) is called an integrating factor of 1) and u(x,y)=C is its primitive. EXAMPLE 3. 3y dx + 2x dy=0 is not an exact differential equation but when multiplied by é(x,y)=x’y, we have 3x?y?dx + 2x°y dy=0 which is exact. Hence, the primitive of 3y dx+2xdy=0 is x’y*=C. See Example 2. If 1) is not exact and no integrating factor can be found readily, it may be possible by a change of one or both of the variables to obtain an equation for which an integrating factor can be found, EXAMPLE 4. The transformation x = t~y, dx = dt-—dy, (i.e., x+y = t), reduces the equation (xty +1)dx + (2x+ 2y +3)dy = 0 to (t+1)(dt-—dy) + (2t+3)dy = 0 or (t+1)dt + (t+2)dy = 0. By means of the integrating factor ; = the equation takes the form + dy + Ltlay = y+ at- tLat=o. t+2 t+2 Then ytt—In(t+2) =C and, since t = xty, 2Qy + x —- In(x+y+2) =C. Note. The transformation xt+ty+1l=t or 2x+2y+3=2s is also suggested by the form of the equation. 12 EQUATIONS OF FIRST ORDER AND FIRST DEGREE 13 A DIFFERENTIAL EQUATION for which an integrating factor is found readily has the form 2) f, (x) 82(y)dx + fo(x):8iCy)dy = 0. By means of the integrating factor a » 2) is reduced to fo (x)* Bo(y) 2') fy (x) dx + &i(y) 4 =0 fF, (x) 8o(y) whose primitive is A100 + £1M) wy -c¢, f(x) Boly) Equation 2) is typed as Variables Separable and in2') the variables are sep- arated. EXAMPLE 5. When the differential equation (3x? y —xy)dx + (2x3y? + xy" dy = 0 is put in the form y (3x? — x) dx + x? (2y’? + y")dy = 0 it is seen to be of the type Variables Separable. The integrating factor =. Vx reduces it to C - 4) ax + (Q2y +y°)dy = 0 in which the variables are sepa- x rated. Integrating, we obtain the primitive 1 2,1 45 _ 3inx+oty + YF C. IF EQUATION 1) admits a solution f(x,y,C) = 0, where C is an arbitrary constant, there exist infinitely many integrating factors é(x,y) such that E(x, y){H (x,y) dx + N(x, y) dy} = 0 is exact. Also, there exist transformations of the variables which carry 1) into the type Variables Separable. However, no general rule can be stated here for finding either an integrating factor or a transformation. Thus we are limited to solving certain types of differential equations of the first order and first degree, i.e., those for which rules may be laid down for determin- ing either an integrating factor or an effective transformation. Equations of the type Variables Separable, together with equations which can be reduced to this type by a transformation of the variables are con- sidered in Chapter 4. Exact differential equations and other types reducible to exact equations by means of integrating factors are treated in Chapter 5. The linear equation of order one 3) x + P(x)sy = Q(x) and equations reducible to the form 3) by means of transformations are con- sidered in Chapter 6. 14 EQUATIONS OF FIRST ORDER AND FIRST DEGREE These groupings are a matter of convenience. A given equation may fall into more than one group. EXAMPLE 6. The equation xdy-—ydx=0 may be placed in any one of the groups since a) by means of the integrating factor 1/xy the variables are separated; thus, dy/y - dx/x = 0 so that Iny -Inx =1nC or yx =C. b) by means of the integrating factor 1/x? or 1/y? the equation is made ex- act; thus, ~<¥Yoi¥% _ 5 and X=C or x dy ~ ydx _ 4 and -~=C,, x? x y? y Y¥__litge, x Cy c) when written as & - ty = 0, it is a linear equation of order one. x Attention has been called to the fact that the form of the primitive is not unique. Thus, the primitive in Example 6 might be given as a) Iny - lnx = 1nC, b) y/x =C, c) y = Cx, d) x/y=K, etc. It is usual to accept any one of these forms with the understanding, already noted, that thereby certain particular solutions may be lost. There is an additional difficulty! EXAMPLE 7. It is clear that y=0 is a particular solution of dy/dx=y or dy~y dx =0, When y #0, we may write dy/y-—dx=0 and obtain Iny-x=1nC with C40; in turn, this may be written as y=Ce*, C#0. Thus, to include all solutions, we should write y=0; y=Ce% C#0. But note that y = Ce”, free of the restrictions imposed on y and C, includes all solutions. This situation will arise repeatedly as we proceed but, as is customary, we shall refrain from pointing out the restrictions; that is, we shall write the primitive as y = Ce*, with C completely arbitrary. In defense, we offer the following observation. Let us multiply the given equation by e~* to ob- tain e~dy — ye"*dx = 0 from which, by integration, we get ey = Cor y = Ce*. In this procedure, it has not been necessary to impose any restric- tion on y or C. CHAPTER 4 Equations of First Order and First Degree VARIABLES SEPARABLE AND REDUCTION TO VARIABLES SEPARABLE VARIABLES SEPARABLE. The variables of the equation M(x,y)dx + N(x,y)dy = 0 are separable if the equation can be written in the form 1) f4(x)+ 8o(y) dx + fo(x): 8, (y) dy = 0. The integrating factor —_i » found by inspection, reduces 1) to the fo(x)* 8o(y) form 81(y) d fy (x) 8oly) from which the primitive may be obtained by integration. For example, (x—1)’ydx + x?(y+1)dy = 0 is of the form 1). The integrat- 2 ing factor —~ reduces the equation to (eb x2y x? dx + YD gy = 0 in which the variables are separated. See Problems 1-5. HOMOGENEOUS EQUATIONS. A function f(x,y) is called homogeneous of degree n if f(Ax, Ay) = XN f(x,y). For example: a) f(x,y) =x’ - xy is homogeneous of degree 4 since F(Ax, Ay) = (Ax)* = (Ax)? Ay) = NM Ot xy) = XM foxy). | b) f(x,y) = e/* + tanz is homogeneous of degree 0 Since Ay y/x el stan Y = 02 4 tan® = 0 f(x,y). Ax x F(Ax, Ay) = x’ + sinx cosy is not homogeneous since F(x, Ay) = Xx? + sin(Ax) cos(Ay) # Xd” f(x,y). c) f(x,y) The differential equation M(x,y)dx + N(x,y)dy = 0 is called homogeneous if M(x,y) and N(x,y) are homogeneous and of the same degree. For example, 2 x in® dx + x arc sin * dy = 0 is homogeneous of degree 1, but neither (x? + y”?)dx = (xy? ~ y*)dy = 0 nor (x + y”)dx + (x-y)dy = 0 isa homogeneous equation. 16 EQUATIONS OF FIRST ORDER AND FIRST DEGREE The transformation y = vx, dy = vdx+xdv will reduce any homogeneous equation to the form P(x, v)dx + Q(x,v)dv = 0 in which the variables are separable. After integrating, v is replaced by y/x to recover the original variables. See Problems 6-11. EQUATIONS IN WHICH #(x,y) AND N(x,y) ARE LINEAR BUT NOT HOMOGENEOUS. a) The equation (a,x +b,y+e)dx + (agxt+tbyy+c,)dy = 0, (a,5,-— a,b, = 0), is reduced by the transformation ayxt by = t, dy = dt — a,dx Db, to the form P(x, t)dx + Q(x, t)dt = 0 in which the variables are separable. See Problem 12. b) The equation (a,x + byy +c) dx + (agx+ bgy+c,)dy = 0, (a,b, —- a,b, # 0), is reduced to the homogeneous form (a,x' + byy')dx!’ + (agx' + bey'’)dy'’ = 0 by the transformation x=x'th, ysyltk in which x=h, y=k are the solutions of the equations ayxtbyyt+e,= 0 and agxtb,yt+c, = 0. See Problems 13-14. EQUATIONS OF THE FORM y. f(xy)dx + x-8(xy)dy = 0. The transformation Zz x dz — z dx —) dy = — x x2 xy = 2, y= reduces an equation of this form to the form P(x,z)dx + Q(x,z)dz = 0 in which the variables are separable. See Problems 15-17. OTHER SUBSTITUTIONS. Equations, not of the types discussed above, may be reduced to a form in which the variables are separable by means of a properly chosen transformation. No general rule of procedure’ can be given; in each case the form of the equation suggests the transformation. See Problems 18-22. SOLVED PROBLEMS VARIABLES SEPARABLE, 1. Solve x°dx + (y+1) dy = 0. The variables are separated. Hence, integrating term by term, 4 t 9 3 o.oo Gg or 3x + 4(y+1) = 3 VARIABLES SEPARABLE 17 v 2. Solve x (y + 1)dx + y’ (x ~1dy = 0. 2 2 The integrating factor 1 reduces the equation to * dx + 2 dy = 0. (y+1)(x-1) x-1 yt+l1 . . 1 1 Then, integrating (x+1+ dx + (y-1+ )dy = 0, x-1 yt¢1 12 1.2 px + x 4+ In(x-1) 4+4y - y+ Infy+]) = Cz, x? + y + 2x ~ 2y+ 2ln(x-l(y+l) = Ci, and (x41)? 4 (y-1)? + 2Inle-D(y+D = C. ¥ 3. Solve 4x dy ~ ydx = x’ dy or ydx + (x? — 4x) dy = 0. . . 1 . dx dy : . The integrating factor => reduces the equation to ———— + — = in which the y(x" - 4x) x(x ~4) y variables are separated, | The latter equation may be written as 4 de - & de + ay = 0 or de - de + 4X =o, x-4 x y x-4 x y Integrating, In(x—4) —Inx+4I1ny =1nC. or (x-4)y" = Cx. dy 4y v4, Solve 2 = ——" or x( -3)dy = 4y dx, dx x(y -3) y y y . . 1 y-3 4 The integrating factor xy reduces the equation to “oy = —dx, x Integrating, y-3 ny = 41nx+I1nC, or y = In(Gx'y*), This may be written as Oxy =e or xy? = Ce’, v 5. Find the particular solution of (1+ x°)dy - xy dx = 0 satisfying the initial conditions x =1, y=2, First find the primitive, using the integrating factor —_t_. y(1+x) 2 Then 9% ~~ dy = 0, iny ~2in(t+x?) = Cy, 3lmy = Im(d+23)+1nC, y2= C142). y 1+%3 3 When x =1, y=2: 2? = C(1+1), C=4, and the required particular solution is y =4(1 +x), HOMOGENEOUS EQUATIONS. / 6. When Mdx + Ndy = 0 is homogeneous, show that the transformation y = vx will separate the variables, When Mdx+Ndy = 0 is homogeneous of degree n, we may write i Mdx +N dy tl ° . x" (My (2) dx + Na(2ydy} = 0 whence My (2) dx ¥ Ny (2ydy The transformation y = vx, dy = vdx + xdv_ reduces this to My (v) dx + Nu(v) fv de + x dv} = 0 or {Mi(v) + vNy(v)}dx + xN,(v) dv = 0 dx Na(v) dv or, finally, — + = in which the variables are separated, x M,(v) + vNa(v) 18 EQUATIONS OF FIRST ORDER AND FIRST DEGREE ; “. Solve (x°+y*)dx — 3xy’dy = 0, The equation is homogeneous of degree 3, We usethe transformation y = ux, dy = vdx + xdv t btai 0 Oph aIN 1) x? {(1403)dx - 30° (vu dx + xdv)} = 0 © 0or (1-2v?)dx - 3v*x dv = 0 in which the variables are separable. 2 dx Upon separating the variables, using the integrating factor 1 —_- 3u dv =0 x(1—-2v) x 4 ~2y3 and Inx+ $ In(l-2v°) = Cy, 2inx + In(1-20°) = nC, or x*(1-2°)=C, Since v = y/x, the primitive is x°(1-2y°/x°) =C or x°- 2y) = Cx. Note that the equation is of degree 3 and that after the transformation x? is a factor of the left member of 1), This factor may be removed when making the transformation, j 8. Solve xdy ~ ydx- x? y? dx = 0. The equation is homogeneous of degree 1. Using the transformation y=vx, dy = vdx + xdv and dividing by x, we have vdx+ xdv—vudx-~VY1-v* dx = 0 or xdv-V1-v* dx = 0. 1 dv dx x When the variables are separated, using the integrating factor —-———_¥, - = 0. xV1—v* 1-v? Then arc sinv—-—Inx=I1nC or arc sin v = In(Cx), and returning to the original va- . pare sin y/x riables, using v = y/x, arc sin = = In(Cx) or Cx = RIS J 9. Solve (2x sinh t + 8y cosh *y dx - 3x cosh t dy = 0. The equation is homogeneous of degree 1. Using the standard transformation and dividing by x, we have 2 sinhv dx -— 3x coshv dv = 0. Then, separating the variables, 2 a _ 3 cosh v v = 0. sinh v Integrating, 2 18x —-31nsinhy = 1nC, x? =Csinhv, and x? = C sink? r. » 10. Solve (2x+ 3y)dx + (y—x)dy = 0. The equation is homogeneous of degree 1, The standard transformation reduces it to (243u)dx + (v -l)\(vdx + xdv) 20 or (v?4 2u42)de + x(v—Ldv = 0. _ d Separating the variables, ds » vol a = de +4 autre gy 2 2m 0. x v? 42042 x y+ 2042 (v41%41 Integrating, Inx + 4 In(v?+ 2v+2) - 2arctan(v+1) = Cy, ln x (v4 2v +2) - 4arctan(v+1) = C, and Ini’ + Qxy + 2x7) -4arctan 772 = ¢, x y ll. Solve (1426 )de + 267/71 - 5 dy = 0. VARIABLES SEPARABLE 19 The equation is homogeneous of degree 0. The appearance of x/y throughout the equation suggests the use of the transformation x-=vy, dx =udy+ ydu, Then (1+ 2e°)(vdy + ydv) + 2e’(1-v)dy = 0, (v + 2e")dy + y(1+ 2e")dv = 0, v and dy + Lee Ww = 0, y v +2e” Integrating and replacing v by x/y, Iny + In(v+2e") = INC and x + aye*!? = C, LINEAR BUT NOT HOMOGENEOUS, +12. Solve (x+y)dx + (3x+3y-4)dy = 0. 0 + The expressions (x+y) and (3x +3y) suggest the transformation x+y We use y= t-x, dy = dt~dx to obtain tdx + (3t-4)(dt-dx) = 0 or (4-2t)dx + (3t-4)dt = 0 in which the variables are separable, 3t-4 oe = ode — 3dt + 2-1 2-t Then 2dx + dt = 0. Integrating and replacing t by x+y, we have 2x -3t-21n(2-t) = Cy, 2-3(x+y)-21n(2-x-y) = Cy, and x+3y+21n(2-x-y) =C. - 13, Solve (2x -5y+3)dx ~ (2x+4y-6)dy = 0. First solve 2x-5y+3 =0, 2x+4y-6=0 simultaneously to obtain x=-h=1, y=k=1, The transformation xex'’t+h=x'+ 1, ax = ax’ yry'tk=y'+ 1, dy = dy’ reduces the given equation to (2x’- Sy’)dx’ — (2x'4+ 4y’)dy’ = 0 which is homogeneous of degree 1. (Note that this latter equation can bewritten down without carrying out the details of the transformation. ) Using the transformation yi=ux', dy’= vdx'+ x' dv, we obtain (2-5v)dx’ — (2+4v)(v dx'+ x’ dv) = 0, (2 -Tv —4v7) dx! — x' (24+ 4v)dv = 0, f and finally ai, fide f 2 ig, x! 3 4-1 3 v+2 Integrating, Inx’ + 5 la(4v ~1) + “an (v+2) = In C, or x? (4u-1)(v +2)° =C, Replacing v by y’/x’, (4y’-x!)(y'+ 2x")? = C, 2 and replacing x’ by x-1 and y’ by y-1, we obtain the primitive (4y-—x-3)(y+2x-3) =C, / 14, Solve (x-y-l)dx + (4y+x-1)dy = 0. Solving x-y-1=0, 4y+x-—-1+=0 simultaneously, we obtain x=h=1, y=k=0. The transformation xex'’t+h=x’+1, ax = dx’ ysy'+k=y' , dy = dy’ reduces the given equation to (x’-—y’)dx' + (4y’+x’)dy’ = 0 which is homogeneous of de- gree 1. (Note that this transformation x-1= x’, y=y¥’ could have been obtained by inspection, that is, by examining the terms (x -—y-—1) and (4y+x-1).) x 20 EQUATIONS OF FIRST ORDER AND FIRST DEGREE Using the transformation y= vx’, dy’=udx'+ x' dv we obtain (1l-v)dx’+ (4v4+ 1)(v dx'+ x’ dv) = 0. f f Then dx! + av +1 dv = dx” + 4 By dv dy = 0, x! 4v2 +1 x! 4v° 41 4v7 +1 2 In x/4+ 5 in(4v? + 1) + S arctan 2v = Cy, In x’ (407 + 1) + arc tan2v = C, ay! 2 2 In(4y’ +x’) + arc tan +- = C, and In[4y? + (x ~1)°] + arc tan x! FORM y f(xy)dx + xgcxy)dy = 0. S15. Solve y(xy+1jdx + x(1+xy+x7y? dy = 0. 2y = C _ x dv —- vdx The transformation wy =v, yrofx, dy = 7 x -—vdkx reduces the equation to =(v +]l)dx 4+ x(l+ve v*) rae ee x which, after clearing of fractions and rearranging, becomes vw? dx — x(1+ v+v*)dv = 0. Separating the variables, we have dx ~ du - du - dv = 0, x p> vy? v 1 1 2 v 12 Then Inx +—+-—-- Inv = (4, 2v° In(-) - 2v ~ 1 = Cu’, 22 x and 2x? y? Iny - 2y-1 = Cx? y?, 16. Solve (y —xy?)dx - (x+ x7 y)dy = 0 or y(l—-xy)dx - x(1l+xy)dy = 0. xdv-vdx The transformation xy=v, ysu/x, dy = 7 reduces the equation to x S(1-v)de = x(1+v) SOUS = 0 or 2Qvadx—x(l+v)dv = 0. x dx 1+ 2 Then 2 - dv = 0, 2inx-Inv-v = 1nC, — = Ce”, and x = Cye™ f 1%. Solve (l-xy+ xy’ dx + (xy ~x?)dy = 0 or y(1—xy + x29?) dx + x(x? y? — xy) dy =0, xdv—-—vdx x? The transformation xy=v, ysu/x, dy = ve v2)de + x(v2—v) LULU = 0 or x x? Then a + (v~ldv = 0, Inn +$v°=~v =C, reduces the equation to vdx + x(v? vidv = 0. and Inx = xy - $x7y? 4 C, uc VARIABLES SEPARABLE 2] MISCELLANEOUS SUBSTITUTIONS, /18, / 19, Solve 2 = (y~4x)) or dy = (y—4x) dx. The suggested transformation y—4x=v, dy=4dx+dv reduces the equation to 4dxe+ dv=v'dx or dx-—% = 09, vp? ~4 Then x+hinvt? og, In pee. InC - 4x, Ute _ Gn and yrae +2 2 on™, v-2 v-2 v—-2 y—4x -2 Solve tan’ (x+y)dx —-dy = 0. The suggested transformation x+y = v, dy = du-dx reduces the equation to tan’v dx - (dv —dx) = 0, dx - —% _ - 9, or dx — cos’v dv = 0. 1+ tan? Integrating, x- bu - ysin 2v = Cy and 2(x-y) = C+ sin 2(x+y). J 20. Solve (2+ ax? yyy dx + (x?y" 4 2)x dy = 0. 2 v Qu 4u" The suggested transformation xy =v, y = =? dy = {we ——dx reduces the equation x x x to ve 2u 4° (2+ 2v)— dx + x(v+2)(— dv —- -——dx) = 0 or v(34+v)dx - x(v+2)duv = 0. x x > dx then 2% 1 we gine 2inv —In(v+3) = ING, = = Cy? (v4.3), x 3.u 3 v0+3 % and 1 = Cyny (2294 3) or ay (xy +3) =C, J 21. 22. Solve (27+ 3y?~ Tx dx — (3x7 +2y?~8)y dy = 0. The suggested transformation x7 = u, y= uv reduces the equation to (2u+ 3v—7)du — (3u+ 2v-8)dv = 0 which is linear but not homogeneous, The transformation u = s+2, v = t+1 yields the homogeneous equation (2s+ 3t)ds — (3s+2t)dt = 0, and the transformation s = rt, ds = rdt+tdr yields Q(r? -1)dt + (2r4+3)t dr = 0. Separating the variables, we have gat + ar+3 a = oat _i dr +3 dr = 0. t r2-]1 t 2rdeil 2r-1 Then 4 Int -In(r+1)+ 51n(r-1) = InC, 4 5 5 5 2 2 5 t - ~t —v- - _ E@ay , wet, eed, way DL , and (x?-y?-1)? = C(x? 4+y?-3). r+l1 s+t u+v-3 x+y? - 3 Solve x (x dx + y dy) + y(x dy —ydx) = 0. Here xdx+ ydy = $d(x*+ y’) and x dy—ydx = x d(y/x) suggest x7 + y¥ = p’, y/x = tan 6, or x=pPcos@, y=psin6, dx = -e sin@d68+cos@dp, dy = P cos8 d6+ sinO dp, The given equation takes the form p’ cos’@ (P dp) + P sin 8 (07 d8) = 0 or dp + tan@ sec@ dO = O, Then 6+ secO = Cj, Vx 4 y? th) = (, and (x? + yy?) (x +1)" = Cx, x 22 EQUATIONS OF FIRST ORDER AND FIRST DEGREE SUPPLEMENTARY PROBLEMS 23. Determine whether or not each of the following functions is homogeneous and, when homogene- ous, state the degree. a) x? xY, xy oD) x+y? xy 2 wey c) d) x+ y cos homo. not homo, homo. y =, homo. x of degree of degree two. zero. of degree one, e) arc sin xy, f) xe!/* ; ye, g) Inx-Il1ny or Ins hy (x? 4 axy + 3y’, 1) x siny +ysinx, not homo, homo. of degree one. homo. of degree zero. homo, of degree one. not homo, Classify each of the equations below in one or more of the following categories: / 24, 25+ 26. al 28. 29. 30. 31. 32. 33. (1) Variables separable (2) Homogeneous equations (3) Equations in which M(x,y) and N(x,y) are linear but not homogeneous (4) Equations of the form y f(xy)dx + x g(xy)dy = 0 (5) None of the above apply. 4ydx+xdy = 0 (14 2y)dx + (4-2x7)dy = 0 y dx—x’ dy = 0 (l+y)dx - (14 x)dy = 0 (xy + y)dx + (x’y ~x)dy = 0 (x sin y x y y y cos dx + % cos = dy = 0 y (x + 2)dx + (x+y) (y dx ~ xdy) = 0 y ae? 4 9? dx — x(x + Vx? + y? dy = 0 (x+y +1)dx + (2x4 2y+1)dy = 0 Solve each of the above equations (Problems 24-32) which fall Ans. 24. 25. 26. 27. xy =C¢ 2 2 1+2 =C (1+ 2y) 3 yx + Cxy (l+y) = C(1+x) + & & Solve each of the following equations. 34. (14 2y)dx - (4-x)dy = 0 Ans. Ans. 28. ¥ = Cxe*?” 29. x sin Y x (1); (1) (1); (1); (4) (2), (5) (2), (3) C 31. Cx — x? +y¥ 32. x +2y + In(xty) =C (2), of degree one (2), of degree two (3) of degree one of degree two in categories (1)-(4). = xX In( vx? + y? ~ x) (x—4)" (14+ 2y) = € VARIABLES SEPARABLE 23 35. xy dx + (14+.x°)dy =0 Ans. y(1+x7) =C 36. cot 8 do + pd = 0 Ans. p =C cos 8 37. (x +2y)dx + (2x4 3y)dy = 0 Ans. a 4 4xy43y =C 38. 2x dy — 2ydx = Vx? +4y? dx Ans. 1+ 4Cy — Cx? = 0 39. (3y- Ix+ W)dx + (Ty-3x+3)dy = 0 Ans. (y-x +1) (y+x-1) = C 40. xy dy = (y+1)(1-x)dx Ans. y+x = InCx(y+1) 41. (y? ~ x? dx + xy dy = 0 Ans. 2x’ y? = x + 42. y(14+ 2xy)dx + x(1-xy)dy = 0 Ans. y = cx? 43. dx + (1~ x )cot y dy = 0 Ans. sin’ y =C = 44. (+ yydx + 3xy’ dy = 0 Ans. x) + 4xy? =C 45, (3x4 2y +1)de ~ (3x + 2y-1)dy = 0 Ans. 1n(15x4+10y~1) + 32 -¥) =C In each of the following, find the particular solution indicated. 46. x dy + 2ydx = 0; when x = 2, y = 1. Ans. xy = 4 47. (74 9? dx +xydy = 0; whenx = 1, y =-1. Ans, x + ax’ y* = 3 48, cosy dx + (l+e )siny dy = 0; whenx=0, y = T/4, Ans. (1+e*)sec y = 2v2 49. (y’ + xy)dx - x’ dy = 0; whenx =1, y = 1. Ans. x = et 7/7 00. Solve the equation of Problem 30 using the substitution y = vx. Ans. xy Inx~y + x - by? 2 Cx? y Bl. Solve y’ = -2(2x+3y)’ using the substitution z = 2x+3y. Ans. 1+ VB(2x +BY) _ ogi * 1 — V3(2x + 3y) 52. Solve (x — 2siny + 3)dx + (2x ~ 4siny -3)cosy dy = 0 using the substitution siny =z. Ans. 8siny + 4x + 9 In(4x - 8Siny +3) =C CHAPTER 5 Equations of First Order and First Degree EXACT EQUATIONS AND REDUCTION TO EXACT EQUATIONS THE NECESSARY AND SUFFICIENT CONDITION that 1) M(x, y)dx + N(x,y)dy = 0 be exact is N 2) om = on . oy Ox At times an equation may be seen to be exact after a regrouping of its terms. The equation in the regrouped form may then be integrated term by term. For example, (x?-—y)dx + (y?-x)dy =0 is exact since OM o 3 ON — = —(x*~y) = -1 = —(y’-x) = =: oy oy Ox Ox This may also be seen after regrouping thus: x’ dx + y’ dy ~ (y dx + xdy) =0. This eqyation may be integrated term by term to obtain the primitive x3/3 + y3/3—xy =C. The equation (y?~x)dx + (x?~y)dy = 0, however, is not exact since on 2y # 2x = on . See also Problem 1. oy Ox IF 1) IS THE EXACT DIFFERENTIAL of the equation wp(x,y) = C, du = Ob ty 4 oF ay = M(x,y)dx + N(x,y)dy. ox oy Then SE ate = M(x,y)dx and u(x,y) = S°W(x, yydx + oy), x where fr indicates that in the integration y is to be treated as a constant and ¢(y) is the constant (with respect to x) of integration. Now oH _ 8 x dp M ao = Sy sy (x, y)dx } + y N(x,y) e from which . = $’(y) and, hence, ¢(y) can be found. See Problems 2-3. INTEGRATING FACTORS. If 1) is not exact, an integrating factor is sought. OM ON dy ax JS f@)dx a) If TT = f(x), a function of x alone, then e is an integrat- ing factor of 1). 24 EXACT EQUATIONS 25 om ON 3 3 dy . . If —— = ~s(y), a function of y alone, then 16 y is an inte- X grating factor of 1). See Problems 4-6. is an integrating factor. See Problems 7-9. b) If 1) is homogeneous and Mx+Ny 40, then Mx+N c) If 1) can be written in the form y f(xy)dx + x g(xy)dy = 0, where f(xy) # 8(xy), then i = i is an integrating factor. xy{ f(xy) — 6(xy) } Mx — Ny See Problems 10+12. d) At times an integrating factor may be found by inspection, after regroup- ing the terms of the equation, by recognizing a certain group of terms as” being a part of an exact differential. For example: GROUP OF TERMS INTEGRATING FACTOR EXACT DIFFERENTIAL dy ~ ydx xdy ~ y dx i x Oy~ YOK. ay x? x? x xdy — ydx 4 ~ Leen ey = d(-2) y y y d’ ady ~ ydx 4 dy de d(1n %) xy x x x dy —ydx dy —ydx 2 xdy - ydx 7 1 “iS = - = d(arc tan 2) x +y xe + y 1+) aayt yes = d{—=+—_}, ifn#1 1 (xy) (n -1) (xy) ~ xdy + ydx A (xy) xdy+yde _ a fincey)}, ifne1 xy Acer ey = d{—_—-1______}, ifngi 1 (x?+ y*) 2(n = 1) (x? + 7?) + wry) zde+ yay _ aft ince? + y2)}, ifn=1 x?4+ y? See Problems 13-19. e) The equation x"y® (my dx + nx dy) + x’y® (uy dx + vx dy)=0, where r,s,m,n, P,0,4,¥ are constants and mv—~nuz#0, has an integrating factor of the form xty4, The method of solution usually given consists of determining a and 6 by means of certain derived formulas. In Problems 20-22, a procedure, essen- tially that used in deriving the formulas, is followed. 26 1. EQUATIONS OF FIRST ORDER AND FIRST DEGREE SOLVED PROBLEMS Show first by the use 2) and then by regrouping of terms that each equation is exact, and solve. 33 4 2 2 2 2 _ dx - dy = a) (4x° y° — Qxy)dx + (38x y - x )dy = 0 d) 2x (ye™ — 1dr + &* dy =0 b) (3e>*y —2x)dx+e*dy = 0 5 3 3 5 6 2 4 4 . c) (cosy + y cosx)dx + (sin x - x sin y)dy = 0 e) (xy + 4x y dx + (3x y + 5x y )dy = 0 OM 32 ON xy a) By 2): — = 12 ~ 2x = — and the equation is exact. Oy x By inspection: (40° y dx + 3x'y dy) — (Qxy dx+ x’ dy) = d(x'y?) - d(x’y) = 0. The primitive is xy? - xy = ¢, b) By 2): oM = 3e* = oN and the equation is exact. oy Ox By inspection: (3e°*y dx + e* dy) - 2x dx = d(e” y) - d(x”) = 0. The primitive is ery ~x° =, c) By 2): om = -~siny + cosx = = and the equation is exact. y x By inspection: (cos y dx — x siny dy) + (y cosx dx + sinx dy) = d(x cos y) + d(y sin x) = 0, Theprimitive is x cosy + ysinx = C. 2 iN d) By 2): = = Qxe~ = on and the equation is exact. x x? ra Pa 2 By inspection: (Qxye" dx + e dy) - 2x dx = d(ye’ ) —d(x') = 0. ar : x 2 The primitive is ye -—~ x = C, e) By 2): ou = 18x°y? + 20x y" = wy and the equation is exact. OY Ox By inspection: (6x y> dx + 3x° y" dy) + (4x3? dx + 5x y' dy) = d(x’y’) + d(x'y’) = 0. The primitive is x°p+ xy = C. Solve (2x° + 3y)dx + (3x+y-1)dy = 0. WM Lg. ON oy Ox and the equation is exact. x Solution 1, Set u(x,y) = J (2x? + 3y)dx = 4x" + 3xy + Ply). 3 Then 5 = 3x+'(y) = Ny) = 3x+y-1, f(y) = y-1,) Oly) = by? y, y and the primitive is $x '+ Bxy + 4y7- y=, or ox + 6xy + y? - 2 =C, Solution 2, Grouping the terms thus 2x dx + ydy ~ dy + 3(y dx+xdy) = 0 and recalling that ydx+xdy = d(xy), we obtain, by integration, tyes by? ~yt+ 3Bxy = Cy as before. EXACT EQUATIONS 27 2 2 3, Solve (ye + 4x?)dx + (2xye*” ~ 3y7)dy = 0 2 2 ol = 2ye*” + Qxyrer? = on and the equation is exact. oy Ox x 2 2 set uxy) = J (ye + 40 )de = 0 + xh + oy), ou xy ' xy 2 2 3 Then 55 = Qxye~ + '(y) = 2Akxye -3y, gy) =-3y, by) =-y, 2 and the primitive is ee +x °~y>=C. 2 2 The equation may be solved by regrouping thus 4x? dx — 3y dy + (ye dx + axye*™ dy) = 0 2 2 2 and noting that ye” dx + axye *” dy = d(e”” ). 4. Solve (x°+y7+x)dx + xydy = 0. Mf ( om = 2y, oN = y; the equation is not exact, oy Ox aM _ ON 3 Ox ~ ] However, oy = 2y~y = i = f(x) and elf (xyde = bbls = e nx x N xy x is an integrating factor. Introducing the integrating factor, we have (+ xy? x’) dx + xy dy = 0 or ade + x°dx + (xy dx + x’y dy) = 0. Then, noting that xy’ dx + xy dy = d(sx7y"), we have for the primitive 4 3 7 + > + bx? y? = Cy or 3x) 4 4x9 + 6x y" = C, 5. Solve (2xy"e% + Qxy? + y)dx + (x’y"e - xy? - 3x)dy = 0. = = Bxye + 2xy'e? + 6xy* + 1, = = oxy" e - Qxy? - 3; the equation is not exact. ry x OM ON However, om _ aw = 8xy°e” + Bxy> + 4 and oy oe 4 -g(y). oy Ox M y Then ofe)ay - etidy/y . .-4iny ing it, the equation takes the form V/y* is an integrating factor and, upon introduc- 2 (2Qxe? + 22 4 “dx + (x? e% - = - 3 =)dy = 0 and is exact. y y y y * y x 1 24 x x Set M(x,y) = J (Qxe° + 2-4 —)dx = xe + — + 2+ Hy). yy? y y Ou 24 x x ! 209 x x ! Then 57 = w eo ~ 384 Ply) = Keo -~ ST - 3s Hy) = 0, Pty) = constant, y y y" y y 2 and the primitive is x’e) + ~— y 28 EQUATIONS OF FIRST ORDER AND FIRST DEGREE 6. Solve (2x°y” + 4x?y + xy” + xy" + 2y)dx + xy + xy + x)dy = 0. iM N = = 4x y + 4x7 4 4xy + 4xy> + 2, E = 2(2xy+1); the equation is not exact. y x om ON 3 3 ? cy OF 2x and the integrating factor is el ede =e, When it is introduced, the N given equation becomes 2 2 (2x? y? + 4x’ y + Quy? + xy" + 2y)e~ dx + ay? + a? y 4 x)er dy = 0 and is exact, 2 x J (2x? 9” + 4x y + Oxy” + xy" + 2y)e~ dx Set H(x,y) = x 2 3 2 x? x 2 x? x oy x? = fo (my + 2 yye~ dx + J (2y + 4x7 y)e dx + J xy'e” dx 22 x° x? ux? = x ye + Qxye + sye + Ply). 9 2 2 a 2 2 Then = = 2x ye* + Qxe” + 2y’e © + Ply) = ay? + xy + x)e~ , P'(y) = 0, ” 2.2 4 x? and the primitive is (2x y + 4xy+y)e° = C. 7%. Show that ae » where Mx+Ny is not identically zero, is an integrating factor of the ho- Ix + NV mogeneous equation M(x,y)dx+N(x,y)dy =0 of degree n. Investigate the case Mx+ Ny=0 iden- tically. We are to show that M + — —dy = 0 is an exact equation, that is, that dix + Ny Mx + Ny 3 M 3 N —(———-) = = Je OY Mx + Ny Ox Max +Ny aM 3M 3N OM 3N (Mx+Ny) = - M(x 4Ne4 yy Ny — ~ MN - My — 3M ) = y oy oy ” yg ” oy ” oy Oy Mx + Ny (Mx + Nyy (Mx + Nyy” and ON OM ON ON OM | fy) == =~ N(x +M — Mx = ~MN ~ Nx = 3 N . (Nx + Ny) Ox (x Ox . vy x) _ * 3x * 3x ox Mx +Ny (Max + Ny) (Mx + Ny)? OM OM ON ON . Nix >+y>) - MOxe~ty>) aM) | BNL ae ay) ae ay) Neal) - Mon) = Oy Mx +Ny Ox Mx + Ny (Mx + Ny)? (Mx + Ny)? (by Euler’s Theorem on homogeneous functions), If Mx+Ny=0 identically, then s = 2 and the differential equation reduces to ydx - xdy = 0 for which l/xy is an integrating factor. 8. solve (x'+ y')dx - xy'dy = 0. The equation is homogeneous and 1 = i is an integrating factor. Upon its intro- Mx + Ny x? EXACT EQUATIONS 29 4 3 duction, the equation becomes C + Td - 7 dy = 0 and is exact. x x x y y* ly’ Set U(x,y) = J (- + Y dx = Inx - = y_ + P(y). x x? 4 xt 3 3 then 2 2 _¥ 4 bity) = -%.+ o'(y) = 0, and the primitive is dy x x? 1 * 4 4 4 Ing - G4 = G or y = 4x Inx+ Cx. x Note. The same integrating factor is obtained by using the procedure of a) above. The equa- tion may be solved by the method of Chapter 4. 9. solve y’dx + (x*° ~ xy - y*)dy = 0. 1 1 The equation is homogeneous and —————- = ~——~-~-—— is an integrating factor, Mx + Ny 2_ 2 y(x"~- y") x? x 2 Upon introducing it the given equation becomes Y dx + AY TY ay = 0 whichis exact, 2 2 2 2 x" -y y(x" - y") x y 1% 1 1 1 x-~y Set , = dx = = - dx = =I1n . et M(x,y) J x? y? 2 J (—< xty 2 xty + oY) 3 x - 1 x 1 Then Oa = = = 5+ by) = ay 7 -S-ss O'M FD OM = Iny, y x? ay y(x?@~ y?) Yo xy y and the primitive is 41n _ + Iny = In, or (x~y)y? = C(x+y). 10. show that me when Mx-Ny is not identically zero, is an integrating factor for the x NY equation Mdx + Ndy = yf,(xy)dx + xfo(xy)dy = 0. Investigate thecase Mx-—Ny=0 identically. The equation AGN + 2 feley) dy = 0 is exact xy {fa (xy) - fo(xy)} xy{fr(xy) -fe(xy)} since py Me pg he fay pha yp Be 2, fa ) ; x(fa fo) ay fax ay : fo dy + fa dy . oy x(fy - fe) x? (fy - fo)? x(f1- fo)? afe _ py ahs _ fe afte _ , Ops 3 fo , ¥(f1- fo) an Poy (= “30? _ fa 3x - fe ox ox “y(fx - fo) y’ (fa - fo)” y(fr-fe) and oft oft ofe Ofe (-y —— + ——) + (y= - x >) ay fi ».2 fo , fa 1S x= hos > ay “x(f, ~ fe) ox y(fi-fe) xy(fs - fo)” This is identically zero since ofy) _ , ofty) . Y 3y - * 3x 30 EQUATIONS OF FIRST ORDER AND FIRST DEGREE If Mx-Ny=0, then 21 =X and the equation reduces to xdy + ydx=0 with solution xy=C. x 11. solve y(x?y? + 2)dx + x(2-2x7y* dy = 0, _ 1 Mx ~Ny 3x3 y3 The equation is of the form yf, (xy)dx + xfg(xy)dy = 0 and is an inte- grating factor. 22 22 x ye 2 ox + 2-2x y d Upon introducing it, the equation becomes y= 0 and is exact, 3x3 y" 3x23 x xy 42 “x 4 2. Set w(x,y) = Jf Ge syd = ff (= + ydx = = Inx + by). 3x3 y? x 3x3 y? 3x2 y? 3 2 2 ~2x°y? 2 Then SE = + OY) = = Py) =- = dy) =- 5 lny, oy 8x? y3 3x7y3 3y 3 2,2 and the primitive is ding ~—- 2 any = int, or x = Cy?e/ * *, 3 3x2 2 3 y The equation may be Solved by the method of Chapter 4. 12. Solve y(2xy +1)dx + x(1+2xy-2xy’)dy = 0. The equation is of the form yf4(xy)dx+ x fo(xy)dy = 0 and 1 — = 1 is an inte- Mx ~ Ny xt yt grating factor, : 1 : 1 1 2 1 Upon introducing it, the equation becomes ( + jdx + ( + ~-)dy = 0 and xy? xtyd wyt x2y3 is exact. x 2 1 1 1 Set ux,y) = Jf ¢ + dx = = - + ply). xdy? xty3 x?y? 3x) y3 3 2 1 2 1 1 then & = + hy) = + -~= by) =-=> Ply) =- Iny, "y x? y3 xy? xdy"t x? ¥3 y y 3.3 and the primitive is ~-Iny — ti. . Cy or y = Ce (x9 +3)/6% "9 - x?y? 3x3 y3 13. Obtain an integrating factor by inspection for each of the following equations. a) (2xy*e? + oxy? + yjdx + (x?y* _ x’y? ~ 3x)dy = 0 (Problem 5) b) (x79? + 2y)dx + (Qe ~ 2 y*)dy = 0 (Problem 11) c) (2xy” + y)dx + (x + Oxy ~ xy? )dy = 0 (Problem 12) a) When the equation is written in the form y" (2xe%dx + x? e dy) + ny dx - xy dy + ydx - 3xdy = 0 the term y! (2xe dx + x’ e dy) = y (an exact differential) suggests that Vy* is a possible integrating factor, To show that it is an integrating factor, we verify that its introduction produces an exact equation. 6) When the equation is written in the form 2(y dx4xdy) + xy dx - 2x y? dy = 0, the term (y dx+xdy) suggests V (xy)® as a possible integrating factor. An examination of the remaining terms shows that each will be an exact differential if k=3, i.e., Vixyy is an integrating factor. 14. 15. 16. 18. 19. EXACT EQUATIONS 31 c) When the equation is written in the form (x dy+ydx) + 2xy(x dy+ydx) - xy dy = 0 the first two terms suggest 1/(xy)®, The third term will be an exact differential if k = 4; thus, VWixy)" is an integrating factor. Solve ydx+ x(1-3x°y")dy =O or xdy+ ydx - 3x y*dy = 0. The terms xdy+ydx suggest 1/ (xy) and the last term requires & = 3. Upon introducing the integrating factor » the equation becomes x dy + y dx - 3 dy = 0 (xy)? xy) y 2,2 whose primitive is a 3Iny=Ci, 6lny = InC - 1 or y? = Cent , a 2x? y2 xy? Solve x dx + ydy + ay? (x? + y)dy = 0. The last term suggests 1/(x?+ y*) as an integrating factor. Introducing it, the equation becomes x dx + ydy + 4y’ dy = 0 and is exact, xray y The primitive is Sin(x’ +y*) + y" = InG, or (x + y ye = C. Solve xdy - ydx - (1-x?)dx = 0. Here 1/x* is the integrating factor, since all other possibilities suggestedby x dy ~y dx render the last term inexact. x dy ee _ (= - 1)dx = 0 whose primitive x x Upon introducing it, the equation becomes Solve (x4x°+ xy? + y')dx + ydy = 0 or xdx + ydy + 4?) dx = 0, An integrating factor suggested by the form of the equation is __1 » Using it, we (x2 4 y2y? xdx+ydy tb ae eg 1 2 2 have ar rey + dx = Q whose primitiveis - > +x =C, or (C+22e)(x +y) = lL (x + 2)? 2(x +y?) Solve x? 2 + xy + V1-x?y? = 0 or x(x dy+ydx) + 1—x?y? dx = 0. d The integrating factor — i reduces the equation tothe form x dy + ydx + & = 0 xv 1—x7y? V1-x?y? whose primitive is arc sin(xy) + Inx = C, 2 3 d xy? = Solve oY 2 Yay Te or (x? + xy -y)dx + (y+ x y+ x)dy = 0, xtxtyt y When the equation is written thus (x? + y’) (x dx+ydy)+xdy-ydx=0, theterms x dy—ydx suggest several possible integrating factors, By trial, we determine V(x? + y?) which reduces 32 20. 21. EQUATIONS OF FIRST ORDER AND FIRST DEGREE x dy ~ydx ~ydx 2 the given equation tothe form xdx + ydy + x dy ~ y ox = xdx+ ydy + —* = 0. 2, 42 ¥.2 uty 1+ (=) x The primitive is bx°+ 4y’ + are tan? = C, or x4 y+2 arc tan Y Cc. x Solve x(4y dx + 2xdy) + y(3y dx + 5xdy) = 0. 8 Suppose that the effect of multiplying the given equation by xy is to produce an equation B atl B+3 +1 8 ar on "lx + 5x y dy) = 0 A) (4x “y dx + aan ey Say) + (3x y each of whose two terms isan exact differential. Then the first term of A) is proportional to B) d(e ty ty 2 ata ry ae + (Bes? y*ay, that is, C) a+2 _ Bl and a ~28=0. Also, the second term of A) is proportional to é 1 D) art yo, . (as iaty ax + (B+ aya yf? ay, that is, E) att. Pts and 5a - 36+ 7. Solving a-28 = 0, 5a—3 = 7 simultaneously, we find a = 2, 6= 1. When these substitutions are made in A), the equation becomes (4x? y? dx + Qnty dy) + (3x y dx + 5x y' dy) = 0, The primitive is xy 4 xy = C Solve (8y dx + 8xdy) + xy (4y dx + 5xdy) = 0. 8B Suppose that the effect of multiplying the given equation by xy is to produce an equation B+1 1 +2 Bt + + A) (8x y dx + 8x" y°dy) + (4x y dx + x 58 ay) = 0 each of whose two terms is an exact differential. Then the first term is proportional to B) d(x t yo") = (at dyarye” de + (B+ 1yxt*t yay, that is, C) a+i1. Bi and a-B=0. 8 8 Also, the second term is proportional to D) ayy =e (assy yas 4 Bray yay, that is, E) a+3 , Ba and = Sa - 48 = 1. Solving a-§ = 0, 5a~48 = 1 simultaneously, we find a=1, B= 1. EXACT EQUATIONS 33 When these substitutions are made in A), the equation becomes (8xy" dx + Bx y dy) + (4x99 dx + 5x y dy) = 0, The primitive is ax’y? a xty? = C. Note. In this and the previous problem it was not necessary to write statements B) and D) since, after a little practice, the relations C) and E) may be obtained directly from A). 22. Solve xy? (2y dx + x dy) ~ (5ydx+%xdy) = 0. 8 Multiplying the given equation by xy , We have + +4 +4 Bt Btl +1 A) (2088 yO te VP yay Sty eae 4 Te Yay) = 0. If the first term of A) is to be exact, then = = ets and a~28= 4. If the second term of A) is to be exact, then a = aa and Ta— 56 = -2, Solving a-28 = 4, Ta-58 =~2 simultaneously, we find a = ~8/3, B = -10/3. - - - - #1 Then, from A), (223 7 ey + wy V5 dy) — (5x 5 V3 oy + Tx 5/5 y 0/5 ayy = 0, each of the two terms is exact, and the primitive is 528. gy. x3 75 ae y -C¢ 53 os 7 5? y + or xy 4 3 M5 SUPPLEMENTARY PROBLEMS 23. Select from the following equations those which are exact and solve. a) (x? ~y)dx ~ x dy = 0 Ans, xy =2°/3+C bb) y¥(x-2y)dx - x’ dy = 0 Cc) (x? 4 y’ dx +xydy =0 d) (x7 + y’ dx + Ixy dy = 0 Ans. xy’ + x? /3 =C e) (x + ycosx)dx + sinx dy = 0 Ans, x4 ay sinx =C 8 f>) ae e ydp + 2pe dé =0 Ans. p(1 + e?”) =C g) dx ~ Va? ~ x? dy = 0 hy) (2x4 B8y+ 4)dx + (3x4 4y+5)dy = 0 Ans, x’ 4 Bxy 4+ 2y° 4 4x 4 SY =C ) (4x79 + 2) dx + (3x"y? - pay = 0 Ans, xy? + In(x/y) =C J) 2Q(u 4 uv)du + (u? + v? dv = 0 Ans. Qu? + 3u7v + vy = C k) (x xe + y? - y)dx + (y (x2 + y? - x)dy = 0 Ans. 2. yy? - 3xy =C 34 25. 26. EQUATIONS OF FIRST ORDER AND FIRST DEGREE Ly) (x+y+1)de - (x-y-3)dy = 0 my) (x+yt+ljdx - (y-x+3)dy = 0 Ans, x 4 oxy - y? + &-by=C n) ese 6 tan @ dr - (r esc 6 + tan? 6)d6 =0 Ans. rcsc@ = In sec@0+C 2 y 1 _ x+y 2 _ o) (y Ey t DH + Gay t PG + DIdy = 0 Ans, In tf @tDy 42)=C 20 2 2 2 2 2 P) (2xye~ * aye + l)dx + (x7 e* yy Qxye~* -2y)dy = 0 Ans. wre pm y =C - Solve the remaining problems above [b), c), &), 1)] using the appropriate procedure of Chap. 4. Ans. 6) x/y = 2 Inx+C g) y= are sinx/fa+ C c) x 4 oxy? = C l) In Yx? + y?~ Qe + 4y 45 ~ are tan 2*= = C For each of the following, obtain an integrating factor by inspection and solve, a) xdx + ydy = (x? + y*)dx Ans. 1/(x + y”) ; xa y? = Ce?* b) (2y—3x)dx + xdy = 0 Ans. x; x’y =x +C c) (x= y’ dx + Ixy dy = 0 Ans, 1/x’; youn In x = Cx d) xdy ~ ydx = 3x? (x? + y’ )dx Ans. Wx ty’); arc tan y/x = x + Cc e) ydx -xdy + Inx dx =0 Ans. 1/x? ; y+ Inx +1=Cx . 2 f) (x7 +y)dx - oxy dy = 0 Ans, 1/x?3 3x ~ y? = Cx g) (xy ~ 2y" dx ~ (x? ~ 3xy)dy = 0 Ans. 1/xy? ; x/y + In(y’ /x”) =C h) (xt+y)dx — (x~y)dy = 0 Ans. 1/(x? + 7); x ay? = Ce? are tan ¥/x 1) 2ydx~ 3xy° dx ~xdy = 0 Ans. x/y’: x /y ~x =C i) ydx + x(x"y=1dy = 0 Ans. y/x?3 By? = 2x7y? = Cx? k) (yt xy + 2x” dx + (x+ 4xy' + By )dy = 0 Ans. 1/(xy +2); In(xy +2)° + x? + 3y" = C For each of the following, obtain an integrating factor and solve. a) xdy ~ ydx = x°e%dx Ans. y = Cx 4 xe” b) (1+ y*)dx = (x4 x°)dy Ans. are tan y = In x/(x4+1) + C€ c) (Qy ~x )dx +xdy = 0 Ans, xy - 2/5 =C d) y'dy + ydx ~ xdy = 0 Ans. yy 4x = Cy e) (By? — xy)dx ~ (x4 exy” dy =0 Ans. 3y* +x In(xy) = Cx f) 3x?" dx + 4 (xy ~3)dy = 0 Ans. xy! - 4y? =C &) yixsy)de - x'dy = 0 Ans. x/y + Inx =C h) (2y 4 Bxy” dx + (x + 2x? yy dy = 0 Ans. xy (14 xy) =C L) yy? ~ 2x" dx + x(2y?— x )dy = 0 Ans. xy? (y?—x°) =C 27. Show that - f(y/x) is an integrating factor of x dy~ydx=0, x CHAPTER 6 Equations of First Order and First Degree LINEAR EQUATIONS AND THOSE REDUCIBLE TO THAT FORM THE EQUATION 1) ov + yP(x) = Q(x), x whose left member is linear inboth the dependent variable and its derivative, is called a linear equation of the first order. For example, Y . 3xy = sinx is called linear while &% + 3xy* = sinx is not. dx dx P(x) dx JP ery dx JP) dx SP ey dk Since 4 (ye! “) y= dy e “) + yP(xje * = e (y + yP).), dx dx dx P(x) dx . . ag . el *) is an integrating factor of 1) and its primitive is y el P a dx ~ Fogel Pe dx +. See Problems 1-7. BERNOULLI’S EQUATION. An equation of the form dy ST YPC) = yO) or mM yen x a P(x) = Q(x) is reduced to the form 1), namely, “ + v{(1—n)P(x)} = (1-n)Q(x), by the xX transformation yrs y cn dy. low, See Problems 8-12. dx l-n dx OTHER EQUATIONS may be reduced to the form 1) by means of appropriate transforma- tions, As in previous chapters, no general rule can be stated; in each in- stance, the proper transformation is suggested by the form of the equation. See Problems 13-18. SOLVED PROBLEMS LINEAR EQUATIONS. 1. Solve dy + Qxy = 4x. dx 2 J Pew) dx = J 2x dx = x? and oP) de = e isan integrating factor, x x x? ox? Then ye = Jaxe*dx = 2 +C or y= 2+Ce™, d 2, Solve xo eye 4 Bx ~ Oe or fy 1 ee oe - 2, dx dx x J P(x) dx = -[% = ~ Inx_ and e mx = i is an integrating factor. x 36 EQUATIONS OF FIRST ORDER AND FIRST DEGREE Then yo = J 2007+ Bx =2)de = | (x+3- =) dx = $274 3x-2Inx+C, 9 or x Qy = x + Ox = 4x Inx + Cx. dy 3 dy 1 2 3. Solve ~2)4% = y+ Wx-2) or = - = 24-2), (e- 2) = yt Bx = oe! ( J P(x) dx = -f = = -In(x-2) and an integrating factor is e Inx~2) = = x~ x~ 1 xe Then yi) = 2) (x-2)- 5 = 2S (x=2)dx = (x-2 +0) or y = (x2) + CQe-2). xe 4. Solve 2 + y cotx = 5e Oe *, Find the particular solution, given the initial conditions: x = 9m, y =-4, An integrating factor is e/C%* 4 . IM 8IMX _ ony ana y sinx = 5) 68 * sinx dx = ~5e 08 * C, When x = 4m, y=-4: (={4)(1) = =5(1) + Cand (= 1, The particular solution is y sinx + be °° * = ol, 2 d - 5. Solve x? dy + (2 3x7 yy = x? or oy + 2 ~ 3x 1, dx dx x3 2 ~3 . J 2 = 3x dx = = i ~ 31nx_ and an integrating factor is 5° x) x? 5 1/x xe 2 Then —, = f—*, = 1, + Cy or 2y = xt Cre xdell/% welt QeWx 6. Solve ~ - 2y cot 2x = 1 - 2 cot 2x - 2 csc 2x. An integrating factor is el? cot 2x dx = @ In sin 2x = ¢cse 2x. Then y csc 2x = f (ese 2x - 2xcot 2x csc 2x - 2 ese” ax)dx = x csc 2x + cot 2x + C or y = x + cos 2x + C sin 2x, 7. Solve ylny dx + (x - Iny)dy = 0. dx The equation, with x taken asdependent variable, may be put in the form — + e@y/yiny) 2 ,in(diny) Then Iny is an integrating factor. Thus, xIny = jiny 2 = siny +k and the solution is 2x Iny = In’y + C, LINEAR EQUATIONS 37 BERNOULLI’S EQUATION. dy 5 -5 dy “4 8 Solve =-y = or — = x, ax y xy y cs y - - d The transformation y * s v, ¥ ° dy -_i@ reduces the equation to dx 4 dx dx - i dv -v=x or dv + 4v = -4x, An integrating factor is eu = ee, 4 dx dx Then ve” = =4 fxe’” dx = ~xe"” + te” + C, yte'* = -xe'* 4 he ** + C, or 1. ~x+ At ce”, ys dy 4 ~4 dy -3 « Solve — + 2xy + = 0 or = + ax = =X, 9 a y+ xy ox Y The transformation y? =U, ~3y"" dy = dv reduces the equation to dv - 6xv = 3x. dx dx dx 2 Using the integrating factor [bx a =e”, we have 2 2 2 2 ve * = f3xe7™ dx = - 1% +C or 1. .t, Ce™ , 2 y3 2 dy 1 1 4 ~u dy 1-3 1 10. Solve =~ + -y = =(1-2% or ~~ + = = =(1-2x). os 37 3° vY y ok 37 3° . +3 -4 dy — dv . dv The transformation =v, -3 — = reduces the equation to —-v = x- 1 y , a & dx for which e ~ is an integrating factor. Then, integrating by parts, ve = {(%&-le dx = -2%e~ ~e+C or 4 = ~-L- 24+ Ce™. y 11. solve 4 y = y (cosx - sinx) or yo oy y y = cosx - sinx, dx dx The transformation yo v, 2 = de reduces the equation to = -v = sinx -cosx for which e is an integrating factor, Then - - - 1 . ve = f(sinx - cos x)e “dx = -e“sinx + C or 77 -sinx + Ce. 3 -3 dy 1 «2 12. Solve xdy ~ {y + xy’(1 4+ Inx)}dx = 0 or ayy? 1+ Inx, . 2 -3 dy dv : dv 2 The transformation =v, -2 — = — reduces theequationto — + —-v =-2(1+1n2x) y Y hk a qd mfx ( for which el? da /x = x? is an integrating factor. Then 2 2 2 43 2 3 x? 23.2 Ux = 92 f(x +x Inx)de = ~ “x°— 3x Inx +C or pe gt iInx) +6, 4 38 EQUATIONS OF FIRST ORDER AND FIRST DEGREE MISCELLANEOUS SUBSTITUTIONS, 13. An equation of the form f'(y) 2 + f(y) P(x) = Q(x) is a linear equation of the first order + vP(x) = Q(x) in the new variable v = f(y). (Note that the Bernoulli equation x yr dy gyre dy y P(x) = Q(x) or (nthy” a y** Gn+1)P(x) = (n+1Q«) is an example. ) d ~y.. . Solve — +1 = 4e ‘sin x or e — +e = 4 sinx, dx dx In the new variable v = f(y) = e, the equation becomes = + v= 4sinx for which e is an integrating factor, Then ve” = 4 fe sinx dx = Qe*(sin x - cosxy+C or e = 2(sinx ~ coSx) + Ce”, . dy 12 : dy _2 14. Solve siny ox = cosx(2cosy —- sin’ x) or -siny at Cos ye cosx) = sin x cosx. x . . dv . 2 In the new variable v = cos y, the equation becomes z + 2vcosx = sin x cosx _ for which er feos x dx = er sinx is an integrating factor. Then sin i i ve T* 5 fers sin’x cosx dx = be? Sin* sin'x - e’° "* sing + se * m4 or cosy = $ sin’x - $ sinx + 4+Ce” Sinn : d j d 1 15. Solve siny &% = cos y(1- x cosy) or Smy oY se Lg, dx cosy dx cos y 1 j 1 Since ot ) = sy, we take v = and obtain the equation dv -v = -X, dy COSY cosy sy a Using the integrating factor e~*, we obtain ve ~ = f=xe “dx = xe +e +C or v= 1 = sec y=xu+1+ Ce, cos y 2 16. Solve x Dy + ary- 2” = 0 or ady ~ ydx+ 3xydx - x dx = 0. Here (x dy -— ydx) suggest the transformation 2 = Ue - . d : : Then x dy ~ y de 3x° Xdx ~ dx = 0 is reduced to x + 3x°v = 1 forwhich e~ is an x x integrating factor, x) wx « =? Thus ve = fe’ dx+C or y =xe fe dx+ Cre”. The indefinite integral here cannot be evaluated in terms of elementary functions. LINEAR EQUATIONS 39 17. Solve (4ar’s - 6)dr + rods = 0 or (rds + sdr) + 3s dr = Sar. r The first term suggests the substitution rs =t which reduces the equation to dt + 34dr = Sar or at 3, . 8, r r dr r r? Then r is an integrating factor and the solution is 3 4 2 3 C tro = rs = 8r°+C or s =~ +—-s 2 rt 18. solve xsin6 gO + (= 2x* cos 9 + cos@)dx = 0 or - zene Bical + 2cos@ dx = x dx. x The substitution xy = cos 6, dy = - z SiO oh cos Oe reduces the equation to x dy dy + Ixy dx = xdx or — + Qy = x. dx 2 An integrating factor is e and the solution is 2 2 2 2 2 ye = cos 6 e =fe xdx = 5° + K or 2cos 0 = x + Cxe. x SUPPLEMENTARY PROBLEMS 19. From the following equations, select those which are linear, state the dependent variable, and solve, a) dy/dx +y = 2+ & ky) y(l+y’)dx = 2(1- 2xy*)dy b) dpf@ + 30 = 2 l) yy! = xy? 4 x = 0 c) dy/dx - y = xy m) xdy-ydx = xv¥x?~y? dy d) xdy —~2ydx = (x-2)e*dx n) $1(t) dx/dt + xdo(t) = 1 e) di/dt - 6i = 10 sin 2t 0) 2dx/dy - x/y + x> cosy = 0 f) dy/dx + y = ye* p) xy’ = yQ-x tanx) + x cos x g) yax + (xy+x- 3y)dy = 0 q) (2+y?)dx - (xy + 2y +y?)dy = 0 h) (2s — e?” yds = 2(se*” — cos 2t)dt r) (1+ y7)dx = (are tan y - x)dy iy xdy + ydx = x y°de s) (2xy’-y)dx + 2x dy = 0 j) adr + (2r cot 6 + sin 20)dO = 0 t) (1+ sin y)dx = [ay cos y ~ x(sec y + tan y)]dy Ans. a) y; IP, e%: y= +Ce” e) i; LP, es iz —4$(3sin 2t + cos 2t) + Ce” b) op: LR, e%: 3p = 2 + Ce" g) x; I.F., yes xy = 3(y-1) + Ce” d) y; I.F., 1/x?: y =e + Cx? j)om LF, sin’ 6; 2r sin?@+sin'@ = C 40 20. 21. 22. EQUATIONS OF FIRST ORDER AND FIRST DEGREE k) x; IF, (lay); (ty yx =2inyty? +C ny xr LP, ef te (Dat/Ar(t), . Soaltdat/dr(t) _ fz Jel dtl) og © a(t) py» LP, ——; y= x cosx + Cx cosx x COS x 24 y7% 4 CY2+y? ~arc tany q) Xx; LF., 1//24 72; x arc tany, r) x: I.F., e x = arc tany - 1 + Ce t) x; I.F., secy+tany; x(secy + tany) =y iC From the remaining equations in Problem 19, Solve those of the Bernoulli type. 2 Ans. c) y=; L/y = 1-x4Ce” Ll) y° = vs y = 14+ Ce” 71 x ~2 -2 : f) y =v, (C+x)ye'+1= 0 0) x =v, x “y=cosy+ysiny+C L) y> =u} 2/y° = (x? + 5x° Ss) y" =U; 3x° = (4x94 Cyy" Solve the remaining equations, h) and m), of Problem 19. Ans, h) 3° - se?’ + sin 2t =C m) y =x sin(y+C) Solve: a) xy! = 2y + xe” subject to y = Q when x = 1, Ans. y = x’ (e*~e) b) L a + Ri. = E sin 2t, where L,R,E are constants, subject to the condition 1=0 when t=0. Ans. 2 = ek sin 2t ~ 2L cos 2t + ale * 7) R°+4L Solve: 2 dy . : . ‘ 3 a) x COSY = = 2x siny +1, using siny =z, Ans. 3x siny = Cx’ 41 b) 4x? yy! = 3x (3y° + 2) + 2(3y" #2), us ing 3y° 4+2=2, Ans. 4x” = (C ~3x°)(3y” +2) c) (xy? yx”) dx + 3xy' dy = 0, using y? = UX. Ans. aye” = xe?” 4 Cx 3 3 2 2 < d) dy/dx +x(x+ty) =x’ (xt+yy - 1. Ans. 1/(x+y) = x +1+Ce e) (y+ ee )dx + (1+ e>)dy = 0. Ans. y+ e = (x+Cye~ CHAPTER 7 Geometric Applications IN CHAPTER 1 it was shown how the differential equation 1) f(x,y,y') = 0 of a family of curves 2) &(x,y,C) = 0 could be obtained. The differential equation expresses analytically acertain property common to every curve of the family. Conversely, if a property whose analytic representation involves the de- rivative is given, the solution of the resulting differential equation rep- resents a one parameter family of curves, all possessing the given property. Each curve of the family is called an integral curve of 1) and particular in- tegral curves may be singled out by giving additional properties, for example, a point through which the curve passes. For convenience, the following properties of curves which involve the de- rivative, are listed. RECTANGULAR COORDINATES. Let (x,y) be a general point of a curve F(x,y) = 0. os 8 iy 3 y i g § gg a) 2 is the Slope of the tangent to the curve at (x,Y). b) + = is the slope of the normal to the curve at (x,y). c) Y-y = DQ y-2) is the equation of the tangent at (x,y), where (X,Y) are the coordinates of any point on it. d) Y~y = + FX -2) is the equation of the normal at (x,7v), where (X,Y) are the coordinates y of any point on it. dx e) x-y and yx are the x~ and y-intercepts of the tangent. Y 41 42 GEOMETRIC APPLICATIONS f) x+y? and yore are the x- and y-intercepts of the normal, gy f/i+ ey and x /1+ 2) are the lengths of the tangent between (x,y) and the x- and y-axes. hy) y / + (2° and x /1+ Gy are the lengths of the normal between (x,y) and the x- Y and y-axes. L) yz and yo are the lengths of the subtangent and subnormal, y J) ds = V (dx). + (ayy + (dy) = dx /1+ Qy - dy /1+ Gy is an element of length of arc. k) ydx or xdy is an element of area, POLAR COORDINATES. Let (0,0) be a general point on a curve p = f(@). gO l) tanwy = 9 = where Y is the angle between the radius vector and the part of the tan- ip gent drawn toward the initial line. 2 d0 m) Pp tanw = p ip is the length of the polar subtangent. dp, n) Pp cot Wy = a is the length of the polar subnormal, . 2d? , . . 0) P sin = a is the length of the perpendicular from the pole to the tangent. Ss g p) ds = V (dey + p° (dey = do/1+ rey = @@ [ey + p is an element of length of arc, q) 5 p40 is an element of area, GEOMETRIC APPLICATIONS 43 TRAJECTORIES. Any curve which cuts every member of a given family of curves at the constant angle w is called an w-trajectory of thefamily. A 90° trajectory of the family is commonly called an orthogonal trajectory of the family. For ex- ample, in Figure (a) below, the circles through the origin with centers on the y-axis are orthogonal trajectories of the family of circles through the origin with centers on the x-axis. (a) (6) In finding such trajectories, we shall use: A) The integral curves of the differential equation f 3) f(x,y, Y—tane , = O 1+ y’tan w are the w-trajectories of the family of integral curves of 1) f(x,y,y') = 0. To prove this, consider the integral curve C of 1) and anw-trajectory which intersect at P(x,y), as shown in Figure (b) above. At each point of C for which 1) defines a value of y’, we associate a triad of numbers (x,y;y’), the first two being the coordinates of the point and the third being the corres- ponding value of y’ given by 1). Similarly, with each point of T for which there is a tangent line, we associate a triad (x,y;y’), the first two being the coordinates of the point and the third the slope of the tangent. To avoid confusion, since we are to consider the triads associated with P as a point on C and as a point on 7, let us write the latter (associated with P on T) as (x,y;y’). Now, from the figure, x=x, y=y at P while y’=tané and y’= tan? are related by tan @ — tanw y’— tan w ; y’ = tan 6 = tan(?-w) = ( ) l+tan¢? tanw 1+Y’tanw Thus, at P (a general point in the plane) on an w-trajectory, the relation y’-~tanw f(x, y,y’) = f(x,y, _————— ) = 0 1+ y’tanw f holds, or, dropping the dashes, f(x,y, Yutane ) = 0. 1+ y’tanw B) The integral curves of the differential equation 4) f(x,y,-l/y’) = 0 are the orthogonal trajectories of the family of integral curves of 1). 44 GEOMETRIC APPLICATIONS C) In polar coordinates, the integral curves of the differential equation 2 do 6, 9? Fy) = 5) F(p, &, Pp are the orthogonal trajectories of the integral curves of dp 6 f(p, 0, —) = 0. ) (p ou SOLVED PROBLEMS 1. At each point (x,y) of a curve the intercept of the tangent on the y-axis is equal to Oxy”, Find the curve, Using e), the differential equation of the curve is y- oh = xy? or ydx ~ x dy = Wax. dx y? 2 + Integrating, ; =x 4 or KZ-xy = Cy, The differential equation may also be obtained di- dy _ y ~ 2xy? | dx x rectly from the adjoining figure as 2, At each point (x,y) of a curve the subtangent is proportional to the square of the abscissa. Find the curve if it also passes through the point (1,e). Using 1), the differential equation is ye = kx? or a = k o » where k is the Y x proportionality factor. Integrating, k1iny = - = +C, When x=1, y=e: k=-1+C and C= kil, 1 The required curve has equation kiny = ~ x t+kee i, 3, Find the family of curves for which the length of the part of the tangent betweenthe point of contact (x,y) and the y-axis is equal to the y-intercept of the tangent. dy.2 dy 2 2 dy From de), we hav x fl + (= = yoxrs or A) = - xy. om g) and e) ave y ne y “hk x y Yh The transformation y = vx reduces A) to 2v du (1+ v*)dx + 2ux dv = 0 or 14 v? & | E Integrating, Inx + In(1t+v’) = InC. 2 Then x(1+ ») =C or ox? + y = Cx is the equation of the family. x GEOMETRIC APPLICATIONS 45 4, Through any point (x,y) of a curve which passes through the origin, lines are drawn parallel to the coordinate axes, Find the curve given that it divides the rectangle formed by the two lines and the axes into two areas, one of which is three times the other, Y P(,y) (a) (5) There are two cases illustrated in the figures. x x x a) Here 3(area OAP) = area OPB, Then af, ydx = xy — S ydx or 4 ydx = xy. To obtain the differential equation, we differentiate with respect to x, dy dy 3y Thus, 4y = tee or ~ =“, y y * Th ax x An integration yields the family of curves y = Cx? x b) Here area OAP = 3(area OPB) and 4, ydx = 3xy. The differential equation is dy = 2, and the family of curves has equation y? = Cx, 3x Since the differential equation in each case was obtained by a differentiation, extraneous solutions may have been introduced. It is necessary therefore to compute the areas as a check, In each of the above cases, the curves found satisfy the conditions, However, see Problem 5, 5. The areas bounded by the x-axis, a fixed ordinate x =a, a variable ordinate, and the partofa curve intercepted by the ordinates is revolved about the x-axis, Find the curve if the volume generated is proportional to a) the sum of the two ordinates, 6) the difference of the two ordinates, a) Let A be the length of the fixed ordinate, The differential equation obtained by differen- tiati Boo, | . 2_ ,dy . = jating 1) nS yrdx = k(y +A) is Ty = kot Integrating, wehave 2) y(C—x) = k. When the value of y given by 2) is used in computing the left member of 1), we find x 2 2 2 3) nf k dx k k @ (C~1nx)* C- mx C- fa = k(y - A). Thus, the solution is extraneous and no curve exists having the property a). x b) Repeating the above procedure with 1'y ms y? dx = k(y —- A), we obtain the differen- tial equation ny? = ae whose solution is 2') y(C— 1x) =k. It is seen from 3) that this equation satisfies 1'). Thus, thefamily of curves 2'y has the required property. 46 GEOMETRIC APPLICATIONS 6. Find the curve such that at any point on it the angle between the radius vector and the tan- gent is equal to one-third the angle of inclination of the tangent, Let @ denote the angle of inclination of the radius vector, 7 the angle of inclination of the tangent, and y the angle between the radius vector and the tangent. Since W = 7/3 = (W+ 0)/3, then W=40 and tanw= tan 30. Using 1), tanW = 9 a6 = tan 30- so that dp. cot 50 dé. dp Pp Integrating, Ine = 21nsins0+1nC, or p=, sin’ 40 = C(1 - cos 8), 7. The area of the sector formed by an arc of a curve and the radii vectors to the end points is one-half the length of the arc, Find the curve, Let the radii vectors be given by 9 = O, and @= @, 8 8 Usi L 2 10 _ i dp 2 2 sing q) and p), 2J,. pd 2p, I +p de Differentiating with respect to @, we obtain the differential equation p= ey +p or =: 1) dp = +pvp?-1 ad. If p= 1, 1) reduces to do =0. It is easily verified that 9 = 1 satisfies the condition of the problem If po #1, we write the equation in the form oe = +dé and obtain the solution pvp -1 sec(C + 9), Thus, the conditions are satisfied bythe circle @=1 and the family of curves sec(C + 0). Note that the families © = sec(C + 0) and e = sec(C ~ @) are the same, DD “ou 8. Find the curve for which the portion of the tangent between the point of contact and the foot of the perpendicular through the pole to the tangent is one-third the radius vector to the point of contact. (a) () 3a = 39 cos(n - WW) = -32 cosW, cos =-1/3, and tan = —2v2. 3a = 30 cos and tanwW = 272, In Figure (a): 9 In Figure (6): g Using 1) and combining the two cases, tanw = dl = +272 or —-=+t+—>- dp P av2 -6 The required curves are the families p = Cell2¥? and p=Ce fav? | GEOMETRIC APPLICATIONS 47 9, Find the orthogonal trajectories of the hyperbolas xy = C, The differential equation of the given family is o + y = 0, obtained by differenti- ating xy = C. The differential equation of the orthogonal trajectories, obtained by replacing dy dx dx — by -—, is -x—+y70 oF dy —~ xdx = 0. dx d dy * yey ~ * Integrating, the orthogonal trajectories are the family of curves (hyperbolas) y- x? = C, Problem 9 Problem 10 2 2 10. Show that the family of confocal conics e + = x = 1, where C is an arbitrary constant, is self-orthogonal, Differentiating the equation of the family with respect to x yields c + ox = 0, where d: ~x, p= oY, Solving this for C, we find C = Ax so that C-X = apy, When these replace- dx x+ Yp x + yp ments are made in the equation of the family, thedifferential equation of the family is found to b owe (x + yp)(px-y) ~ Ap = 0. Since this equation is unchanged when p is replaced by -1/p, it is also the differential equation of the orthogonal trajectories of the given family, 11. Determine the orthogonal trajectories of the family of cardiods p = C(1+ sin @). . . ge . . . 1 Differentiating with respect to @ to obtain dp = C cos 6, solving for C = de , ag cos 7 | d@ and substituting for C in the given equation, the differential equation of the given family *8 dP _ pecos@ | dO 1+ sin@ The differential equation of the orthogonal trajectories, obtained by replacing < by - 2d is dp de _ cos O or dp + (sec @ + tan@)d@ = 0. dp e(1+ sin@) Pp Then Inf + In(sec@ + tan9@) - Incos@ = InC or p = Ecos = C(1-sin@). sec 6 4 tan @ 48 GEOMETRIC APPLICATIONS 12. Determine the 45° trajectories of the family of concentric circles x7+y’=C. 13. 14. equation by ~————--——_ = The differential equation of the family of circles is xt+yy’= 0. The differential equation of the 45° trajectories, obtained by replacing y’ in the above a ° a a y’ - tan45 y 1 is xt yd 1 1+ y’tan 45° 1+y’ 1+y! = 0 or (x+y)dy + (x-y)dy = 0. Using the transformation y =vx, this equation is reduced to dx v+l + (v2 +1)dx + x(v+l)dv = 0 or = dv = 0. x e+ Integrating, Inx + 4 in(v?+ 1) + arctanv = In ky, In x?(1+ v7) = In kK - 2 arc tanv, - ti and Paar y? = Ke 2 are tan yx . . 2 ~26 6 In polar coordinates, the equation becomes 9 = Ke or Pe =C, SUPPLEMENTARY PROBLEMS Find the equation of the curve for which a) the normal at any point (x,y) passes through the origin. Ans, xy? =C b) the slope of the tangent at any point (x,y) is 4 the slope of the line from the origin to the point, Ans, y? = Cx c) the normal at any point (x,y) and the line joining the origin to that point form an isos- celes triangle having the x-axis as base. Ans, y? x? =C d) the part of the normal drawn at point (x,y) between this point and the x-axis is bisected by the y-axis. Ans, y?+2x?=C e) the perpendicular from the origin to a tangent line of the curve is equal to the abscissa of the point of contact (x,y). Ans. x?+y" = Cx f) the arc length from the origin to the variable point (x,y) is equal to twice the square root of the abscissa of the point. Ans. y = t(are sinvx + ¥x=2x2)+C g) the polar subnormalis twice the sine of the vectorial angle. Ans. p = C ~ 2 cos@ h) the angle between the radius vector and the tangent is $ the vectorial angle. Ans. p = C(1 - cos 0) . 8 t) the polar subtangent is equal to the polar subnormal. Ans. p = Ce Find the orthogonal trajectories of each of the following families of curves. ayx+2y=C Ans. y-2x = K f) y =x-1+Ce” Ans, x = y-1+ Ke” b) xy = C xy? = K g) y= 2x7(1- Cx) x4 3y? In(Ky) = 0 c) xe yay? =€ y = Kx? h) p = a cos @ o = 6 sin®€ d) y = Ce"** y? = 24K i) p = a(1+sin@) po = b(1- sin @) eyy =x /C-x) ty?) = Kx? 49?) jy p = asec + tang) p= be -~siné CHAPTER 8 Physical Applications MANY OF THE APPLICATIONS of this and later chapters willbe concerned with the mo- tion of a body along a straight line. If the body moves with varying velocity v (that is, with accelerated motion) its acceleration, given by dv/dt, is due to one or more forces acting in the direction of motion or in the opposite direction. The net force on the mass is the (algebraic) sum of the several forces. EXAMPLE 1. A boat is moving subject to a force of 20 pounds on its sail and a resisting force (lb) equal to 1/50 its velocity (ft/sec). If the direc- tion of motion is taken as positive, the net force (lb) is 20~—v/50. EXAMPLE 2, To the free end of a spring of negligible mass, hanging verti- cally, a mass is attached and brought to rest. There are two forces acting on the mass -— gravity acting downward and a restoring force, called the spring force, opposing gravity. The two forces, being opposite in direction, are equal in magnitude since the mass is at rest. Thus, the net force is zero. Newton’s Second Law of Motion states in part that the product of the mass and acceleration is proportional to the net force on the mass. When the sys- tem of units described below is used, the factor of proportionality is k=1 and we have mass x acceleration = net force, THE U. S. ENGINEERING SYSTEM is based on the fundamental units: the pound of force (the pound weight), the foot of length, and the second of time. The derived unit of mass is the slug, defined by mass in slugs = weight in pounds | é in ft/sec? Hence, mass in slugs x acceleration in ft/sec*® = net force in pounds, The acceleration g of a freely falling body varies but slightly over the earth’s surface. For convenience in computing, an approximate value g = 32 ft/sec”? is used in the problems. SOLVED PROBLEMS 1. If the population of a country doubles in 50 years, in how many years will it treble under the assumption that the rate of increase is proportional to the number of inhabitants? Let y denote the population at time t years and yo the population at time t=0. Then dy dy 1) dt = ky or 7 = kdt, where k is the proportionality factor. First Solution, Integrating 1), we have 2) Iny = kt + In€ or y= Ce”, At time t=0, y=yo and, from 2), C= yo. Thus, 3) y= yoe™ 49 50 PHYSICAL APPLICATIONS At t=50, y=2Yoe From 3), 2o = Yoe” or eo * = 2, kt Ort When y = 3yo, 3) gives 3=e . Then 3° =e ht t eSokt ( = 2 and t = 79 years, Second Solution. Integrating 1) between the limits t=0, y=yo and t=50, y= 2yo, 50 2%, dy f — = k f dt, In 2¥9 — In yo = 50k and 50k = In 2, Yo y 0 Integrating 1) between the limits t=0, y=y¥q and t=t, y =3y, 3y. t 0 f dy = ef dt, and ln 3 = kt. % 9% 0 Then 50 1n3 = 50kt = t In2 and ¢t = was = 99 years. n 2. In a certain culture of bacteria the rate of increase is proportional to the number present, (a) If it is found that the number doubles in 4 hours, how many may be expected at the end of 12 hours? (b) If there are 10* at the end of 3 hours and 4-10° at the end of 5 hours, how Many were there in the beginning? Let x denote the number of bacteria at time t hours, Then 1) = = kx or < = kdt. a) First Solution, Integrating 1), we have 2) Inx =kt + 1I1nC or x= Ce*, Assuming that x = x9 at time t=0, C =x) and x = noe’. At time t = 4, x = 2x9. Then 2% = xoe’” and e*” = 2, When t = 12, x = xen” = xo (e”) = %9(2°) = 8x9, that is, there are 8 times the original number. Second Solution, Integrating 1) between the limits t=0, x=x_ and t=4, x= 2%, 2% dx %q ox y = kf at, In 2% - 1n X% = 4k and 4k = In 2, 0 Integrating 1) between the limits t=0, x =x 5 and t=12, x =x, u x 12 f de k f dt, and In = 12k = 3(4k) = 3 1n2 = 1n8, % * 0 xo Then x = 8x9, as before, . . 1 b) First Solution, When t = 3, x = 10". Hence, from 2), 10° = Cee* and C = ~ . 4 4 k 4-10" ° When t = 5, x = 4°10. Hence, 4°10 = Ce? and C = oe 10° _ 4+10° 2k h Equating the values of C, —— = Eo Then e = 4 and e = 2, eo © 10° 10° Thus, the original number is C = ——— = -—— bacteria. aR 8 PHYSICAL APPLICATIONS 5] Second Solution. Integrating 1) between the limits t=3, x=10" and t=5, x=4-10", 410" dx 5 f, — = ef dt, In 4 = 2k and k = 1n 2, 10 x 3 Integrating 1) between the limits t=0, x=x9 and t=3, x= 10", y 0 3 4 4 f dx = ef dt, in 22 = 32 - 31n2 = 1n8 and x = 1O as before, % x ( xo 8 3, According to Newton’s law of cooling, the rate at which a substance cools in moving air is proportional to the difference between the temperature of the substance and that of the air. If the temperature of the air is 30° and the substance cools from 100° to 70° in 15 minutes, find when the temperature will be 40°. Let T be the temperature of the substance at time t minutes. Then aT = +~k(T ~ 30) or aT = -kdt. dt T ~ 30 (Note, The use of —k here is optional. It will be found that k is positive, but if +k is used it will be found that k is equally negative, ) Integrating between the limits t = 0, T = 100 and t = 15, T = 70, 70 a) f aT = —k f dt, In 40 - In 70 = ~15k = In 4 and 15k = In 7. 0.56. 100 T - 30 0 7 4 Integrating between the limits t = 0, T = 100 and t = t, T = 40, 40 t 6 f aT = -k f dt, In 10—1n70=-kt, 15kt=151n7, t= 15 In’ = 52 min, too T - 30 0 0.56 4, A certain chemical dissolves in water at a rate proportional tothe product of the amount un- dissolved and the difference between the concentration in a saturated solution and the con- centration in the actual solution. In 100 grams of a saturated solution it is known that 50 grams of the substance are dissolved. If when 30 grams of the chemical are agitated with 100 grams of water, 10 grams are dissolved in 2 hours, how much will be dissolved in 5 hours? Let x denote the number of grams of the chemical undissolved after t hours, At this time the concentration of the actual solution is 30 - x and that of a saturated solution is =. Then dx - + dx k Oe px( 2d. B0R%) = py 2% or a re dt 100 100 100 x x + 20 5 Integrating between t = 0, x = 30 and t = 2, x = 30-10 = 2, 20 20 2 dx & dee. Ef at, and k = 2 In® = 0.46, 30 & 30 x+ 20 5 % 2 6 Integrating between t = 0, x = 30 and t = 5, x =x, x x 5 f dx f dx = « f dt, In-——2% = k = -0.46, x, 370046 30 x 30 x +20 5 Yo 3(x + 20) x+20 5 = 0.38, and x = 12, Thus, the amount dissolved after 5 hours is 30-12 = 18 grams, n2 5. PHYSICAL APPLICATIONS A 100 gallon tank is filled with brine containing 60 pounds of dissolved salt, Water runs into the tank at the rate of 2 gallons per minute and the mixture, kept uniform by stirring, runs out at the same rate. How much salt is in the tank after 1 hour? Let s be the number of pounds of salt in the tank after t minutes, the concentration then being s/100 lb/gal. During the interval dt, 2dt gallons of water flows into the tank and 2dt gallons of brine containing = dt = 5 dt pounds of salt flows out. Thus, the change ds in the amount of salt in the tank is ds = — a dt. + ~t/50 re : ~t/50 Integrating, s = ¢ % . At t = 0, s = 60; hence, C = 60 and s = 60e When t = 60 minutes, s = 60e°> MW 60(.301) = 18 pounds. The air in a certain room 150'x50'x12' tested 0.2% CO,. Fresh air containing 0.05% CO, was then admitted by ventilators at the rate 9000 ft}/min, Find the percentage CO, after 20 minutes, Let x denote the number of cubic feet of CO. in the room at time t, the concentration of CO, then being x/90,000. During the interval dt, the amount of CO, entering the room is 9,000(.0005)dt ft? and the amount leaving is 9,000 * dt ft, : . : x x-45 Hence, the change dx in the interval is dx = 9,000(.0005 — jdt = = dt. 90,000 10 : ~t/10 Integrating, 10 In(x ~ 45) =-t + I1nC, and x = 45 + Ce . ~t/10 At t = 0, x = .002(90,000) = 180. Then C = 180-45 = 135 and x = 45 + 135e tA . tes. - ~2 . 63 Woen t = 20, x = 45 + 135e = 63. The percentage CO, is then = .0007 = 0.07%. 90,000 Under certain conditions the constant quantity Q calories/ second of heat flowing through a wall [— 125 om— is given by aT Q = —-kA—, dx where k is the conductivity of the material, A(cm?) isthe area of a face of the wall perpen- dicular to the direction of flow, and T is the temperature x(cm) fromthat face such that T de- creases as x increases, Find the number of cal- ories of heat per hour flowing through 1 square meter of the wall of a refrigerator room 125 cm thick for which k = 0.0025, if the temperature of the inner face is -5°C and that of the outer : °° face is 75 C, direction of flow 100 cm Let x denote the distance of a point within the wall from the outer face. Integrating dT = —- A a fromx = 0, T= 75 to x = 125, T=-5, ~5 125 2 fo ar = -2 fac, 0 = Serasy, and gq = SOM = 896.0025)0009" 1g cal 5 kA 4p kA 125 125 sec Thus, the flow of heat per hour = 3600Q = 57,600 cal. PHYSICAL APPLICATIONS 33 8. A steam pipe 20 cm in diameter is protected with a covering 6 cm thick for which k = 0,0003. (a) Find the heat loss per hour through a meter length of the pipe if the surface of the pipe is 200°C and that of the outer surface of the covering is 30°C. (b) Find the temperature at a distance x > 10 cm from the center of the pipe. At a distance x >10 cm from the center of the pipe, heat is flowing across @ cylindrical shell of surface area 21x em? per cm of length of pipe. From Problem 7, Q = -kA dT = _onkx 2 or ank dT = -Q e, dx dx x a) Integrating between the limits T = 30, x = 16 and T = 200, x = 10, 200 10 TR 2rtk f aT = -Q dx 340k = Q(1n 16 —-1n10) = Q1nl.6 and Q= 340 cal/sec. 30 16% 1n 1.6 Thus, the heat loss per hour through a meter length of pipe is 100(60)7Q = 245,000 cal. b) Integrating 2%kdT = ~ ae de between the limits T = 30, x = 16 and T=T, x = x, ni.6 x n 170 * dx 170 x 170 16 f dT = — f =, T-30 =- In~ and T= (30+ In—)°c, 30 In 1.6 yy * Inl.6 16 Inl6 Check. When x =10, T= 30+ — In 1.6 = 200°C. When x=16, T = 30 + 0 = 30°C. ni, 9, Find the time required for a cylindrical tank of radius 8 ft and height loft to empty through a round hole of radius 1 inch in the bottom of the tank, given that water will issue from such a hole with velocity approximately v = 4.8/h ft/sec, h being the depth of the water in the tank, The volume of water which runs out per second may be thought of as that of a cylinder 1 inch in radius and of height v. Hence, the volume which runs out in time dt sec is m(ty(4.gvh)dt = —~(4.8vh)dt. 12 144 Denoting by dh the corresponding drop in the water level in the tank, the volume of water which runs out is also given by 64%dh, Hence, —(4.gVh)dt = —64ndh or dt ~ £4(144) dh _ igo HF 144 4.8 Vp Vk Integrating between t = 0, h= 10 and t = t, h=0O, t 0 gh 0 f dt = —1920 f — >, and t= -seov [ = 3840V10 sec =3 hr 22min. 0 10 VA 10. A ship weighing 48,000 tons starts from rest under the force of a constant propeller thrust of 200,000 lb. a) Find its velocity as a function of time t, given that the resistance in pounds is 10,000v, with v = velocity measured in ft/sec. 6) Find the terminal velocity (i.e., v when t-—+o) in miles per hour. (Take g = 32 ft/sec?, ) 11. 12. PHYSICAL APPLICATIONS Since mass (slugs) x acceleration (ft/sec’) = het force (1b) = impetus of propeller — resistance, then 48,000(2000) dv _ a9 900 — 10,0000 or 1 ye. 2, 32 dt dt 300 300 Integrating, ve (1300 = 20 42/500 dt 20¢°/ °° + C, 300 ~£/30 - 0 a) When t= 0, v= 0; C=-20 and v = 20 — 2007/7? - aoc — 9%), b) As t+, v = 20; the terminal velocity is 20 ft/sec = 13.6 mi/hr. This may also be ob- : . tage d tained from 1) since, as v approaches a limiting value, a + 0. Then v = 20, as before. t A boat is being towed at the rate 12 miles per hour, At the instant (t = 0) that the towing line is cast off, a man in the boat begins to row in the direction of motion, exerting aforce of 20 1b, If the combined weight of the man and boat is 480 lb and the resistance (lb). is equal to 1,75v, where v is measured in ft/sec, find the speed of the boat after 3 minute, Since mass (slugs) x acceleration (ft/sec’) tt net force (1b) = forward force ~ resistance, then 480 dv _ ng | 15 y or dv 7) 4, 32 dt dt 60 3 Integrating, ve 1/60 - sfen™ dt = ae + C¢ When t = 0, v = =o" - 88. ¢. >= and » = 82 4 216 ,-7%/60° (60) 5 7 35 When t= 30, v = 20 4 218,-35 7 35 = 11,6 ft/sec. A mass is being pulled across the ice on a sled, the total weight including the sled being 80 lb, Under the assumption that the resistance offered by the ice to the runners is negli- gible and that the air offers a resistance in pounds equal to5 timesthe velocity (v ft/sec) of the sled, find a) the constant force (pounds) exerted on the sled which will give it a terminal velocity of 10 miles per hour, and 6b) the velocity and distance traveled at the end of 48 seconds. Since mass (slugs) x acceleration (ft/sec”) = net force (1b) = forward force - resistance, then Oa sr Suv or dy + Qu = 2 Fe where F (1b) is the forward force. 32 dt dt 5 ~ ~2t Integrating, v = = + Ce *Y When t=0, v=0; then C = -5 and A) v = Aa ~ ev), ao a) AS t+@, Fey = 10(5280) _ 44, The required force is F = 2 ip, 5 (60)" 3 b) Substituting froma) in A), v = Fa ~e*), 48 6 48 _ ? = S et/sec, and s = J vdt = = f (l-e at oe = 697 ft. 0 When t =48: v = Sane wl uw PHYSICAL APPLICATIONS 13. A spring of negligible weight hangs vertically. A mass of m slugs is attached to the other end. If the mass is moving with velocity vg ft/sec when the spring is unstretched, find the velocity v as a function of the stretch x ft. According to Hooke’s law, the spring force (force opposing the stretch) is proportional to the stretch. tt Net force on body weight of body -— spring force. dv dv dx dv . Then m— = mg — kx or m—— = mvu— = mg — kx, since — =v, at dx dt dx at . 2 2 Integrating, mv = Qmgx — kx + C, When x = 0, v = ug. Then C = mu, and mv? = 2mgx ~ kx? + mue. 14. A parachutist is falling with speed 176 ft/sec when his parachute opens. If the air resis- tance is Wv?/ 256 lb, where W is the total weight of the man and parachute, find his speed as a function of the time t after the parachute opened. : Net force on system = weight of system — air resistance. 2 W W Then ~ dv = - ov or de => = at . g dt 256 v? — 256 8 Integrating between the limits t = 0, v = 176 and t=t, v=v, t v Uv -1 f _ du = = i { dt, i in v 6 = _f . 176 y*% 986 8 Jo 32 v +16 1176 Bho v ~ 16 5 16 5 6i5e In -~in2 = —4t, vate es fet and lv = 1p SHPE v +16 6 v +16 6 6-5e7"" Note that the parachutist quickly attains an approximately constant speed, that is, the terminal speed of 16 ft/sec. 1h. A body of mass m slugs falls from rest in a medium for which the resistance (1b) is propor- tional to the square of the velocity (ft/sec). If the terminal velocity is 150 ft/sec, find a) the velocity at the end of 2 seconds, and b) the time required for the velocity to become 100 ft/sec. Let v denote the velocity of the body at time t seconds. Net force on body = weight of body ~ resistance, and the equation of motion is dv 2 1 m— = — Kv*, ) at me . 2 Taking g = 32 ft/sec, it is seen that some simplification is possible by choosing K = 2mk’. d Then 1) reduces to de = 2(16 - ae) or A = -2 dt, at kv ~ 16 Integrating, In kv-4 _ ~16kt + InC or kv-4 _ Ce ORE, kv+4 kv +4 When t = 0, v = 0. Then C=-1 and 2) ku-4 _ ae Rt ku+4 When to, v = 150. Then en tOkt _ 0, k= 2 and 2) becomes ~ 7150 ae BE 15 v +150 16. PHYSICAL APPLICATIONS a) When t= 2, U—190 - 278° _ _ 493) and v = 61 ft/sec. v + 150 b) When v = 100, e °-.2 = e © and ¢ = 3.7 sec, A body of mass m falls from rest in a medium for which the resistance (1b) is proportional to the velocity (ft/sec). If the specific gravity of the medium is one-fourth that of the body and if the terminal velocity is 24 ft/sec, find (a) the velocity at the end of 3 sec and (b) the distance traveled in 3 sec, Let v denote the velocity of the body at time t sec. In addition to the two forces acting as in Problem 15, there is a third force which results from the difference in specific grav- ities. This force is equal in magnitude to the weight of the medium which the body displaces and opposes gravity. Net force on body = weight of body -— buoyant force -— resistance, and the equation of motion is du 3 m— = mg ~ —mg — Kv = —=mg ~ Kv, dt 4 4 Taking g = 32 ft/sec® and K 3mk, the equation becomes = 3(8-—kv) or dv = 3dt. 8—kv Integrating from t= 0, v=0 to t=t, v=uv 1 v t -3kt - 5 in(8-kv)], = 3th : ~1n(8—kv) + In8 = 3kt, and kv = 8(1-e ). When to, v = 24, Then k= 1/3 and 1) v = 24(1~e7°). a) When t = 3, v = 24(1 - e7°) = 22.8 ft/sec. b) Integrating v = = = 24(1 -e7"y between t = 0, x = 0 and t = 3, x =x t x at [3 -3 x 0 = 24(t +e ) 0 and x = 24(2+ e “) = 49.2 ft. The gravitational pull on a mass m at a distance s feet from the center of the earth is pro- portional to m and inversely proportional to s*. a) Find the velocity attained by the mass in falling from rest at a distance 5R from the center to the earth’s surface, where R = 4000 miles is taken as the radius of the earth. 6) What velocity would correspond to a fall from an infinite distance, that is, with what velocity must the mass be propelled vertically up- ward to escape the gravitational pull? (All other forces, including friction, are to be neg- lected. ) The gravitational force at a distance s from the earth’s center is km/s*, To determine k, note that the force is mg when s = R; thus mg = km/R? and k = gR*, The equation of motion is dv n— = = =e dt 2 ns WL gy BR - nek or vdo - gk? 2, dt ds 1) ve ds 3? s? the sign being negative since v increases as s decreases, a) Integrating 1) fromv = 0, s=5R tov=v, s=R, v 2 rf ds 12 21 1 4 2. 8 vdv = -gR . $v? = gR*(>-—) = =eR, = =(32) (4000 )(5 280), J g S 2 2 Bh Ga) 28 ve = (32) Xe and v = 2560V165 ft/sec or approximately 6 mi/sec. 18. PITYSICAL APPLICATIONS b) Integrating 1) from v = 0, s+a@ to v= R ; 2 ds eR’ f SS, 4? = a 52 v 5 vdv = 2eR, v = 6400/33 ft/sec or One of the basic equations in electric circuits is Le 4 Ri = 1 dt E(t), where L(henries) is called the induc- Eo™ tance, R(ohms) the resistance, t(am- peres) the current, and E(volts) the electromotive force or emf. (In this book, R and L will be constants.) a) Solve 1) when E(t) =EKEo and the ini- tial current is tp. a 6) Solve 1) when L = 3 henries, R = 15 (a) ohms, E(t) is the 60 cycle sine wave of amplitude 110 volts, andzi=0 when t=0. a) Integrating L <= + Ru= Eo, eh dE = pelt dt = Eo Pt/i +C t Eo =i9g-— R t = Fo/R, a constant. . fo . ~Rt/L When t=0, 2 =U. Then C and R (l-e Rt/ Note that as t+o, wa wJ approximately 7 mi/sec. L E(t) R () or ou = Bo , ce Ft, _ = Rt/L ) toe . b) Integrating 3 + 15t = Eo sinwt = 110 sin 2n(60)t = 110 sin 120Nt, t . 1 11 t i mt - 120% cos 120Nt ie?! = HO po! sin 120nt dt = i110 .? 5 Sin l2ont = 1207 cos 120n¢ 120 0 —_ 0 Cc 3 25 + 14400" . 22 sin 120"%t - 24m cos 120nt ~5t or toe SS ee lt (Ce . 3 1+ 576" When t =0, 1=0. Then C = 220 24m 3(1 + 5767) 22 sin 120nt - 24n cos 120nt + 24ne7” and GO i et 3 1+ 5767? A more useful form is obtained by noting that the sum of the squares of the coefficients of the sine and cosine terms is the denominator of the fraction above. Hence, we may define tT sing = =. and cos f = =; (1 + 57617) (1+ 5767”) -5t . 2 Tt so that i= 2 i (cos ¢ sin 120nt ~ sin cos 120nt) + aire . 3(1 + 57677) 1+ 5767 -5t tT = 22 Z Sin(zont - P) 1i6ne - 3(1 + 5767)? 1+ 576%

S-ar putea să vă placă și